Craniofacial Anomalies Flashcards
A 7-year-old girl presents to the clinic with poor school performance, speech difficulties, and hypernasality. The patient also has a history of immunodeficiency and hypocalcemia. Clinically, there is no evidence of overt or submucosal cleft palate. Which of the following is the most appropriate next step for her care?
A) Formal speech evaluation
B) MRI
C) Palatoplasty
D) Observation
The correct response is Option A.
Velopharyngeal dysfunction can be divided into insufficiency, incompetence, and mislearning. The term insufficiency is related to a structural defect that results in poor closure of the velopharyngeal port. Incompetence is usually associated with neurological or muscular causes to inefficient port closure. Mislearning is less frequent and related to normal anatomy and muscle function, but inadequate production of sounds.
The patient described has many of the medical abnormalities observed in 22q11.2 deletion syndrome. These include, but are not limited to, congenital heart defects, hypocalcemia, immune deficiencies, speech delay/disorders, palatal abnormalities, intellectual disabilities, and psychiatric disorders. Late speech disorder diagnosis is not uncommon and warrants a thorough evaluation in a cleft center; this patient would also benefit from a genetics consultation. Depending on the formal speech evaluation results, further measures of velopharyngeal closure, such as nasometry and pressure flow, as well as imaging with videofluoroscopy and nasopharyngoscopy, might be recommended.
A 4-month-old boy is brought to the emergency department by his parents for evaluation after he had a seizure for the first time. Physical examination shows low nasal bridge and short neck. On laboratory studies, serum calcium level is 6.0 mg/dL (normal = 9.0 - 10.5 mg/dL) and serum phosphate level is 7.0 mg/dL (normal = 2.5 - 4.5 mg/dL). Which of the following is the most likely diagnosis?
(A) Albright syndrome
(B) Binder syndrome
(C) Carpenter syndrome
(D) Down syndrome
(E) Ectodermal dysplasia
The correct response is Option A.
Albright syndrome is the only syndrome listed that affects calcium and phosphate metabolism. It is caused by an autosomal-dominant mutation affecting receptor binding to adenylate cyclase. The resultant craniofacial malformations include a low nasal bridge and a short neck. Binder syndrome is manifested by maxillary hypoplasia and midface retrusion. Carpenter syndrome is a rare autosomal-recessive condition that results in brachycephaly due to variable synostoses, in addition to defects of the cardiovascular, musculoskeletal, and genital systems. Down syndrome is caused by trisomy of the 21st chromosome. Craniofacial manifestations include brachycephaly, a low nasal bridge, and inner epicanthal folds. Ectodermal dysplasia is an X-linked recessive syndrome that results in hypoplastic skin and sparse dermal appendages.
A 23-year-old man comes to the office because of a progressive 15-year history of severe unilateral volume loss in the face. The patient says he is unhappy with the changes in his facial appearance but has not noted any changes recently. On examination, volume loss is significant and appears to be limited to the soft tissue. No bony deficit is noted. Which of the following is the most appropriate management?
A) Alloplastic bony augmentation of the maxilla
B) Autologous soft-tissue augmentation with a cross-facial nerve graft followed by microvascular gracilis muscle transfer
C) Autologous soft-tissue augmentation with a microvascular parascapular flap
D) Autologous soft-tissue augmentation with a pedicled temporalis muscle flap
E) Hyaluronic acid augmentation of the soft tissues
The correct response is Option C.
It is most likely that this patient has progressive, but now stable, hemifacial atrophy, or Parry-Romberg syndrome. The course is characterized by progressive unilateral atrophy of the soft tissues of the face. The underlying skeleton may be affected in severe forms of the disease. Surgery is indicated when the changes stabilize. The recommended treatment is free tissue transfer, preferably a microvascular parascapular flap, followed by a secondary autologous fat grafting for refinement. Augmentation of the maxilla is not indicated in the absence of bony atrophy. Staged cross-facial nerve grafting followed by microvascular gracilis transfer is indicated for absence of facial nerve function. A pedicled temporalis muscle flap would likely not provide sufficient bulk where needed and potentially leave a hollow at the donor site. Alloplastic augmentation of the soft tissues with hyaluronic acid is only approved for volume loss due to human immunodeficiency virus infection.
The 2-year-old boy shown has features typical of which of the following syndromes?
(A) Apert
(B) Binder
(C) Crouzon
(D) Pfeiffer
(E) Treacher Collins
The correct response is Option E.
The appearance of the patient shown demonstrates Treacher Collins syndrome, characterized by bilaterally symmetrical abnormalities of structures within the first and second branchial arches. Features include a convex facial profile with a retrusive lower jaw and chin, anti-mongoloid slant of the palpebral fissures, lower lid colobomas, partial absence of eyelid cilia, absent or malformed external ears, hypoplasia of the malar bones, and variable cleft palate.
Binder syndrome is characterized by nasomaxillary hypoplasia.
Apert, Crouzon, and Pfeiffer syndromes are all craniosynostotic syndromes that result in midfacial hypoplasia and a concave facial profile of varying degrees.
A 5-year-old boy has marked malar hypoplasia, a class II anterior open bite, and clockwise rotation of the occlusal plane. There is hypoplasia of the thumbs bilaterally. Which of the following is the most likely diagnosis?
(A) Bilateral craniofacial microsomia
(B) Goldenhar syndrome
(C) Klippel-Feil syndrome
(D) Nager syndrome
(E) Treacher Collins syndrome
The correct response is Option D.
This child has findings most consistent with Nager syndrome, or acrofacial dysostosis, an autosomal recessive disorder characterized by hypoplasia of the orbits, zygoma, maxilla, mandible, and soft palate. Although the findings are similar to Treacher Collins syndrome (an autosomal dominant disorder), patients with Nager syndrome can also exhibit preaxial limb anomalies and mental retardation. Auricular defects may be present.
Patients with bilateral craniofacial microsomia typically have hypoplasia of the mandibular ramus and varying degrees of auricular hypoplasia. Patients with Goldenhar syndrome have hemifacial microsomia and epibulbar dermoids.
Klippel-Feil syndrome is characterized by significant fusion of the cervical spine with varying involvement of the thoracic and lumbar spinal regions. Affected patients have limited cervical motion and a low posterior hairline.
A 12-hour-old male newborn is evaluated in the neonatal intensive care unit. A diagnosis of CHARGE syndrome is suspected. Review of the medical record and physical examination show congenital heart defect, eyelid colobomas, microtia, hypogonadism, and growth retardation. Which of the following additional conditions is/are most likely in this patient?
A) Craniosynostosis
B) Intrahepatic hemangiomas
C) Lower lip pits
D) Orofacial cleft
E) Syndactyly
The correct response is Option D.
Cleft lip and/or palate is found in up to 20% of patients with CHARGE syndrome (Coloboma of the eye, heart defects, atresia of the nasal choanae, retardation of growth and/or development, genital and/or urinary abnormalities, and ear abnormalities and deafness). It is the second most common syndrome associated with cleft lip and palate, following van der Woude syndrome. CHARGE syndrome is believed to be caused by a microdeletion or mutation in the CHD7 (chromodomain helicase DNA-binding protein 7) gene. Over the years, there have been other conditions found to be associated with this syndrome, including limb anomalies, renal anomalies, omphalocele, cranial nerve anomalies, and tracheoesophageal fistula.
Syndactyly is not typically associated with CHARGE syndrome.
Intrahepatic hemangiomas are not typically associated with CHARGE syndrome.
Craniosynostosis is not typically associated with CHARGE syndrome.
Lip pits are commonly found in patients with van der Woude syndrome.
Which of the following arteries is most likely to be injured in a patient undergoing Le Fort I osteotomy?
A) Descending palatine
B) External carotid
C) Greater palatine
D) Internal carotid
E) Internal maxillary
The correct response is Option A.
Descending palatine artery is the correct response since it is at greatest risk after pterygoid osteotomy and down fracture. It is easily visible after down fracture and mobilization of the Le Fort I segment. It is typically injured during osteotomy prior to its division into the greater and lesser palatine arteries.
External carotid artery is incorrect because it is within the neck. Its branches ascend in the face and end in the sphenopalatine artery as it enters the pterygoid foramen.
Internal maxillary artery is incorrect because it is the larger artery prior to division into multiple smaller branches including the middle meningeal, sphenopalatine, and descending palatine arteries.
Greater palatine artery is incorrect because the greater and lesser palatine arteries are below the level of the pterygoid osteotomy. The descending palatine is more proximal and is at greater risk for injury.
The anterior fontanelle typically closes completely at how many months of age?
(A) 3
(B) 9
(C) 12
(D) 24
(E) 36
The correct response is Option D.
The fontanelles are nonossified membranous intervals in the skull located at the angles of the parietal bones in infants. The anterior and posterior fontanelles are located in the midline, and the anterolateral (sphenoid) and posterolateral (mastoid) fontanelles are found laterally.
The anterior fontanelle is the largest, measuring approximately 4 cm anteroposteriorly and 2.5 cm transversely. It is located at the junction of the sagittal, coronal, and frontal sutures. The posterior fontanelle is triangular in shape and is found at the junction of the sagittal and lambdoid sutures. The sphenoid and mastoid fontanelles are irregular, small fontanelles that correspond to the sphenoid and mastoid angles of the parietal bones, respectively. Although the posterior fontanelle typically closes by age 2 months, the anterior fontanelle does not completely close until between the ages of 18 and 24 months.
A 12-week-old girl has a cleft of the soft palate, symptoms of respiratory obstruction when lying supine, and history of poor weight gain. Which of the following is the most likely diagnosis?
A) Hemifacial microsomia
B) Isolated cleft palate
C) Pierre Robin sequence
D) Van der Woude syndrome
E) Velocardiofacial syndrome
Correct answer is option C.
The child described has Pierre Robin sequence, which is associated with the symptoms of micrognathia, glossoptosis, and respiratory distress. Children with this condition often have difficulty with feedings as well. Pierre Robin sequence is believed to result from the tongue coming between the palatal shelves in the developing fetus. These patients are managed conservatively with positioning and tube feedings if necessary. If, however, the airway cannot be protected with positioning, then surgical management includes lip-tongue adhesion, distraction osteogenesis, and if necessary, tracheostomy.
Hemifacial microsomia is manifest by underdevelopment of the middle and external ear, zygoma, maxilla, and facial muscles and often is associated with a Tessier No. 7 cleft (macrostomia).
Isolated cleft palate does not cause airway obstruction or micrognathia but can be associated with slow weight gain and difficulty feeding.
Van der Woude syndrome is associated with cleft lip and palate and lower lip pits from accessory salivary glands.
Velocardiofacial syndrome is the most common syndrome seen in association with cleft lip and palate. Other clinical manifestations associated with this syndrome include velopharyngeal insufficiency, facial and cardiac anomalies, and absence of the thymus and parathyroid glands.
A 14-year-old girl is brought to the office by her parents for consultation regarding facial asymmetry. On the basis of the photograph shown, which of the following is the most likely diagnosis?
(A) Congenital cranial (VII) nerve palsy
(B) Craniofacial microsomia
(C) Goldenhar syndrome
(D) Romberg disease
(E) Unilateral coronal craniosynostosis
The correct response is Option D.
The patient shown has Romberg disease (progressive hemifacial atrophy). This is a unilateral condition that affects girls more commonly than boys, and onset is in childhood with progression from 2 to 10 years of age. It usually involves skin, subcutaneous tissue, muscle, and bone but spares the cranial nerves and their function. Treatment is complex, with reconstruction of facial bone structure and augmentation followed by cutaneous contour correction with free tissue transfer. Parascapular flap is the flap of choice.
Unilateral coronal craniosynostosis can result in incomplete correction of vertical dystopia, secondary to continued growth constriction at the affected side cranial base. Soft-tissue loss and lower face asymmetry are not associated features.
Hemifacial microsomia is a congenital, nonprogressive abnormality of the first branchial arch derivatives (ie, mandible and auricle). The mandibular ramus and condyle are variably hypoplastic, and overlying soft tissues (muscle, subcutaneous fat) are often hypoplastic. Ipsilateral macrostomia (transverse facial cleft) can also present in this syndrome.
Goldenhar syndrome is a variant of craniofacial microsomia and is distinguished by presence of concomitant ocular abnormalities, including epibulbar dermoid.
In the Tessier system, which of the following classifications represents the most common facial cleft?
A) Tessier No. 0
B) Tessier No. 3
C) Tessier No. 7
D) Tessier No. 9
E) Tessier No. 14
The most common facial cleft is a Tessier No. 7. This is a cleft that begins at the lateral oral commissure and extends laterally. From a soft-tissue standpoint, it creates macrostomia. A Tessier No. 0 cleft involves the midline of the upper lip and nose. The Tessier No. 14 cleft is the cranial extension of this. A Tessier No. 3 cleft involves the lateral nasal ala and the medial canthus of the eye. A Tessier No. 9 cleft is actually the least common cleft. It extends from the superolateral orbit into the temporal region.
A 7-year-old boy has hypernasality and velopharyngeal incompetence. He underwent repair of a ventricular septal defect at birth and repair of a cleft palate at age 9 months. His mother says that he has had difficulties with language learning. Physical examination shows upward slanting of the palpebral fissures, a broad nasal root, a small mouth, and a thin upper lip.
Which of the following studies is most likely to lead to a diagnosis in this patient?
(A) Measurement of serum creatine kinase level
(B) Chromosomal karyotyping
(C) Fluorescent in situ hybridization (FISH) analysis
(D) MR angiography
The correct response is Option C.
This 7-year-old boy has findings consistent with velocardiofacial syndrome, or Shprintzen syndrome, an autosomal dominant disorder with variable expressivity. Affected patients characteristically have velopharyngeal insufficiency, developmental delay, and facial abnormalities, including upward slanting of the palpebral fissures and a prominent nose with a broad nasal root and narrow alar base. Velocardiofacial syndrome has been diagnosed in as many as 8% of children with clefts of the secondary palate; however, cleft palate is not always seen in children with velocardiofacial syndrome. Cardiac anomalies, such as aberrant carotid arteries, are present in most patients, and the risk for arterial bleeding is increased during pharyngeal flap repair.
Because velocardiofacial syndrome is associated with a deletion on the long arm of chromosome 22q11.2, fluorescent in situ hybridization (FISH) analysis can be used to test any child with suspected velocardiofacial syndrome for chromosomal deletions. Although routine chromosomal karyotyping will not detect the small deletion associated with velocardiofacial syndrome, fluorescent-tagged DNA probes can be used in conjunction with routine cytogenetic examination. On examination with a fluorescent microscope, a child with a normal chromosomal pattern will have two signals (one on each chromosome), whereas only one signal will be present in the child with velocardiofacial syndrome. -
The diagnosis of 22q11.2DS is suspected when clinical symptoms are present. The diagnosis is confirmed by a blood test that can detect a microscopic chromosomal deletion on chromosome 22. There are many new tests to detect this deletion including whole genome array, SNP array, comparative genomic hybridization, and MLPA. FISH studies have also been useful in finding the deletion in most patients. Furthermore, routine chromosome (cytogenetic) testing is also performed because a small number of affected individuals have a chromosome rearrangement involving chromosome 22q which may change the recurrence risk counseling for the parents.
Serum creatine kinase levels are increased in patients with muscular dystrophy. Although MR angiography will show aberrant blood vessels in patients with velocardiofacial syndrome, it is not diagnostic.
Synonyms
Autosomal dominant Opitz G/BBB syndrome
Cayler cardiofacial syndrome
conotruncal anomaly face syndrome
DiGeorge syndrome
Shprintzen syndrome
Velocardiofacial syndrome
22q11.2 deletion syndrome
In infants with hemifacial microsomia, which of the following deformities should be corrected initially?
(A) Abnormalities of the temporomandibular joint abnormalities
(B) Auricular deformities
(C) Deficiency of the mandibular ramus
(D) Deficiency of the mandibular body
(E) Macrostomia
The correct response is Option E.
Hemifacial microsomia is a condition associated with embryologic abnormalities involving the first and second branchial arches, including a Tessier No. 7 cleft. It is characterized by incomplete development of the external and middle ear, mandible, zygoma, maxilla, temporal bone, parotid gland, tongue, and the muscles of the palate and those that control mastication and facial expression. Affected patients have multiple clinical manifestations such as macrostomia, first branchial cleft sinus, and abnormalities of the cranial nerves.
Multiple procedures are necessary to reconstruct the face in patients with hemifacial microsomia. Macrostomia repair should be performed within the first few months after birth, similar to a typical cleft lip repair. Skeletal and soft-tissue repair is indicated at age 5 to 6 years. The mandibular deformities are typically corrected first, followed by the maxillary defects. Correction of soft-tissue deformities is generally delayed until skeletal reconstruction has been completed.
Which of the following is the most common cause of scaphocephaly without craniosynostosis?
A) Fibromatosis colli
B) Hereditary dolichocephaly
C) Ocular torticollis
D) Prematurity
E) Ventriculoperitoneal shunt
The correct response is Option D.
Positional plagiocephaly is very common since the American Academy of Pediatrics began the ‘back to sleep’ campaign to decrease sudden infant death syndrome in 1992. Occipital flattening is the most common type seen. Positional plagiocephaly risk factors include prematurity, intrauterine position, congenital muscular torticollis, plural birth, motor delays, and hypotonia. Ventriculoperitoneal (VP) shunts may induce a secondary craniosynostosis if over-shunting occurs. Scaphocephaly head shape without craniosynostosis is noted in the premature babies in the NICU due to positioning and tone. This corrects with growth, increased tone, and repositioning. Presence of a VP shunt does not lead to the head shape of scaphocephaly without craniosynostosis but may be seen in children after intraventricular hemorrhage in the NICU. Scaphocephaly without craniosynostosis does occur in older infants and children but is not known to be a hereditary condition. Both ocular torticollis and congenital muscular torticollis can cause a head tilt and are associated with deformational plagiocephaly. Fibromatosis colli is seen with severe muscular torticollis and is associated with plagiocephaly.
A 4-month-old boy is brought to the emergency department by his parents for evaluation after he had a seizure for the first time. Physical examination shows low nasal bridge and short neck. On laboratory studies, serum calcium level is 6.0 mg/dL (normal = 9.0 €“10.5 mg/dL) and serum phosphate level is 7.0 mg/dL (normal = 2.5 - 4.5 mg/dL). Which of the following is the most likely diagnosis?
(A) Albright syndrome
(B) Binder syndrome
(C) Carpenter syndrome
(D) Down syndrome
(E) Ectodermal dysplasia
The correct response is Option A.
Albright syndrome is the only syndrome listed that affects calcium and phosphate metabolism. It is caused by an autosomal-dominant mutation affecting receptor binding to adenylate cyclase. The resultant craniofacial malformations include a low nasal bridge and a short neck. Binder syndrome is manifested by maxillary hypoplasia and midface retrusion. Carpenter syndrome is a rare autosomal-recessive condition that results in brachycephaly due to variable synostoses, in addition to defects of the cardiovascular, musculoskeletal, and genital systems. Down syndrome is caused by trisomy of the 21st chromosome. Craniofacial manifestations include brachycephaly, a low nasal bridge, and inner epicanthal folds. Ectodermal dysplasia is an X €‘linked recessive syndrome that results in hypoplastic skin and sparse dermal appendages..
A 3-year-old boy is brought to the office because of a congenital soft tissue notch of the lower lateral eyelid. Which of the following is the most appropriate Tessier classification for the underlying craniofacial cleft?
A) Tessier No. 3
B) Tessier No. 4
C) Tessier No. 6
D) Tessier No. 10
The correct response is Option C.
The Tessier No. 3 cleft defect symptoms include a defect between the lateral incisors and canine. This cleft typically involves the alar base and medial canthal region. In severe cases, the cleft may enter the orbit medial to the punctum. Tessier No. 4 facial clefts extend from the upper lip, around the alar base, along the nasomaxillary junction, and across the tear duct and medial orbital tissues. Tessier No. 6 facial clefts may disrupt bone and soft tissues along the lower lateral orbit. These defects are frequently associated with colobomas of the lower eyelid. Extending into the frontal bone, Tessier No. 10 clefts involve the superior orbital rim and medial third of the orbit. Proptosis with fronto-orbital encephalocele may be present with significant clefts at this location. No. 7 clefts may produce macrostomia and extend through the lateral zygomatic arch. An image is shown.
An 11-year-old girl who is undergoing evaluation because of the appearance of her nose. Examination shows a short, flattened nasal bridge and midface hypoplasia. The anterior nasal spine is absent on radiographs. Which of the following is the most likely diagnosis?
(A) Binder’s syndrome
(B) Goldenhar’s syndrome
(C) Nager’s syndrome
(D) Treacher Collins syndrome
(E) Velocardiofacial syndrome
The correct response is Option A.
This 11-year-old girl has Binder’s syndrome which is characterized by localized nasomaxillary hypoplasia resulting in a flat nasal bridge and a short, retracted columella. The columella and upper lip are depressed into the floor of the nose, and the anterior nasal spine is absent. Angle class III malocclusion is usually present.
Goldenhar’s syndrome, or oculoauriculovertebral dysplasia, involves asymmetry of the hard and soft tissues of the face. This condition is most often unilateral but may be seen bilaterally in some patients. Manifestations of this syndrome include hypoplasia involving the mandible and underlying soft tissues of the face, epibulbar dermoids, and varied degrees of microtia on the affected side. Most patients have associated vertebral abnormalities.
Nager’s syndrome, or acrofacial dysostosis, is an autosomal recessive disorder characterized by craniofacial and upper extremity abnormalities. Patients with Nager syndrome have hypoplasia of the orbits, zygoma, maxilla, mandible, and soft palate. Auricular defects may also be present. Hypoplasia or agenesis occurs in the radius, thumbs, and metacarpals. Some patients may have radioulnar synostosis and elbow joint deformities.
Patients with Treacher Collins syndrome, or mandibular dysostosis, have hypoplasia of the zygoma, maxilla, and mandible, downward slanting of the palpebral fissures, colobomas of the lower eyelids, absence of eyelashes, and auricular defects.
Velocardiofacial syndrome is characterized by overt or submucous clefting of the palate and cardiac abnormalities. Most patients have abnormal facial features, including narrow palpebral fissures and a prominent nose with a square nasal root and narrow alar base. The anterior nasal spine is present.
A 3-year-old boy is brought to the office because of abnormal head shape since birth. Photographs are shown. The patient is developmentally appropriate and has no other medical problems. Which of the following is the most appropriate surgical procedure for correction of this deformity?
A) Bilateral fronto-orbital advancement
B) Endoscopic craniotomy and helmet therapy
C) Monobloc distraction
D) Spring-mediated cranioplasty
E) Total vault reconstruction
The correct response is Option E.
The patient in the photograph has the classic features of sagittal suture synostosis. The primary clinical features of scaphocephaly are lengthening of the cranial vault in the anterior-posterior dimension, an anteriorly displaced cranial vertex, bullet-shaped occiput, biparietal and/or temporal narrowing and frontal bossing. All of these features are present in this patient. Because of the patient’s late clinical presentation for treatment, the most appropriate surgical procedure would be a total cranial vault reconstruction of which there are many variations and techniques.
Because of the patient’s age, he is not a candidate for either endoscopic-assisted wide strip craniotomy or spring-mediated cranioplasty. Ideal candidates for either procedure are ideally under age 6 months and 9 months, respectively, for an adequate clinical result. The results of both techniques have been shown to be comparable to traditional open remodeling procedures and are generally less invasive surgical procedures.
Although the monobloc distraction may allow the opportunity to remodel the forehead, there is no clinical indication for midface distraction in the patient described. A monobloc distraction alone will not correct the other abnormal features of scaphocephaly. It is not an indicated procedure for the correction of isolated sagittal suture synostosis.
Bilateral fronto-orbital advancement may allow the opportunity to remodel the forehead, but it will not address the other cranial vault abnormalities. There is no indication for remodeling the supraorbital bar in this case.
Scaphocephaly is associated with which of the following suture synostoses?
(A) Bilateral coronal
(B) Lambdoid
(C) Metopic
(D) Sagittal
(E) Unilateral coronal
The correct response is Option D.
Scaphocephaly is just one form of craniosynostosis, or varying deformities of the cranial vault resulting from restrictions in development. Detectable craniosynostotic abnormalities occur in one of approximately 1800 neonates. The coronal, lambdoid, metopic, and sagittal sutures are primarily involved; minor sutures can include the frontonasal, frontosphenoidal, and temporosquamosal. Mental retardation is often seen in patients with multiple synostotic sutures; this occurs as a result of prolonged restriction of brain growth and cranial vault development secondary to fusion of the overlying sutures.
Scaphocephaly is associated with synostosis of the sagittal suture. Affected patients have an elongated, narrow cranial vault. In contrast, patients with bilateral coronal synostosis have brachycephaly, in which the frontal portion of the skull is wide and flat and the cranium is shortened anteroposteriorly. Lambdoid suture synostosis is also known as occipital or posterior plagiocephaly. This condition, characterized by an oblique posterior flattening, is rarely seen congenitally and more often results from childbirth or prolonged head positioning, such as during sleep. Infants with metopic suture synostosis, or trigonocephaly, have a triangularly shaped forehead with decreased bitemporal distance. Unilateral coronal synostosis, or frontal plagiocephaly, involves oblique frontal flattening of the skull.
A 48-year-old man is evaluated after a motor vehicle crash, and CT scanning of the head shows a moderately displaced anterior/posterior frontal sinus fracture. The images are shown. The patient also has associated rhinorrhea; the fluid is sent for analysis. Which of the following is the most sensitive diagnostic finding that would indicate treatment of the fracture with cranialization and surgical repair of the dural tear?
A) High bacterial contamination
B) High glucose content
C) Low hemoglobin percentage
D) Positive beta-2 transferrin level
E) Salty postnasal drainage
The correct response is Option D.
A frontal sinus fracture involving the anterior and posterior tables with displacement greater than one table width and associated cerebrospinal fluid (CSF) leak mandates treatment with cranialization and repair of the dural tear. While minimally displaced posterior table fractures with CSF leak may be observed for spontaneous resolution of the leak, any significant displacement will likely not allow the dural tear to adequately heal and significantly increases the risk for bacterial contamination and meningitis.
Accurately diagnosing the presence of a CSF leak in conjunction with significantly displaced posterior table frontal sinus fractures is important in deciding the ultimate management of these complex injuries. The beta-2 transferrin level is the most sensitive test for diagnosis of a true CSF leak. While CSF fluid does tend to have a high glucose content, this could also be seen in bloody rhinorrhea as well and is not as sensitive as a beta-2 transferrin test. Likewise, although CSF rhinorrhea often presents as a salty tasting postnasal drainage, this finding is more subjective and has poor sensitivity. Low hemoglobin concentrations are unlikely after acute fractures but can also be present without CSF leak. High bacterial contamination of the rhinorrhea does not correlate with CSF leak, but the risk for bacterial contamination of the meninges leading to meningitis is the reason for aggressive treatment of displaced posterior table frontal sinus fractures with associated CSF leak.
According to the Tessier classification, which of the following clefts is most closely associated with macrostomia?
(A) No. 1
(B) No. 3
(C) No. 5
(D) No. 7
(E) No. 9
The correct response is Option D.
The Tessier classification of craniofacial clefting was first proposed in 1973. This system integrates both tissue findings and underlying skeletal deformities; embryopathogenesis is not considered. According to the Tessier system, the No. 7 cleft manifests as macrostomia and absence of the zygomatic arch. This common sporadic cleft, which has variable expressivity, is more likely to be seen in male neonates and occurs in one of every 3000 births.
The Tessier No. 1 cleft lies just lateral to the midline, beginning at the cupid’s bow and passing through the dome of the nostril lateral to the anterior nasal spine. Notching of the alar dome is a distinctive feature. The nasal bone may be absent, but the septum is unaffected. Hypertelorism and encephalocele may also be associated.
The Tessier No. 3 cleft involves the orbit. It begins in the alveolus between the lateral incisor and canine and extends through the maxilla into the lacrimal bone. This cleft is commonly referred to as a naso-ocular cleft because the inferomedial wall of the orbit is absent. Associated soft-tissue deformities include shortening of the nose, colobomas of the nasal alae and the lower eyelids medial to the punctum, obstruction of the nasolacrimal duct, malformation of the lower canaliculus, and hypoplasia of the medial canthal tendon.
With the rare Tessier No. 5 cleft, the cleft moves laterally and becomes oblique. It begins beneath the canine and extends through the maxillary sinus to the orbital floor. Colobomas of the lateral lower eyelids and clefting of the upper lip medial to the oral commissure are associated.
The Tessier No. 9 cleft is merely a supraorbital extension of the Tessier No. 5 cleft.
Which of the following is the most common craniofacial anomaly?
(A) Bilateral craniofacial microsomia
(B) Goldenhar syndrome
(C) Hemifacial microsomia
(D) Nager syndrome
(E) Treacher Collins syndrome
The correct response is Option C.
Hemifacial microsomia is the most common craniofacial anomaly; affected patients have unilateral malformations of the mandibular ramus and facial paralysis. Mandibular growth is impaired, leading to inadequate development of the mandibular and osseous complex on the involved side.
Bilateral craniofacial microsomia is less frequent than hemifacial microsomia.
Goldenhar syndrome, a variant of hemifacial microsomia, is characterized by vertebral or hemivertebral anomalies, lipodermoids, and epibulbar dermoids. The facial abnormalities resemble a Tessier No. 7 cleft. This condition may be bilateral.
Nager syndrome is an autosomal recessive disorder that manifests as hypoplasia of the orbits, zygoma, maxilla, mandible, and soft palate, as well as hypoplasia or agenesis of the radius, thumbs, and metacarpals. Auricular defects may also be associated.
Treacher Collins syndrome is an autosomal dominant disorder that involves hypoplasia of the zygoma, maxilla, and mandible. The palpebral fissures have an antimongoloid slant and the lashes of the medial lower eyelids are absent. Colobomas of the lower eyelids and malar defects are also associated. The preauricular hair may be displaced, and micrognathia may be present. Findings are similar to the Tessier No. 6, No. 7, and No. 8 clefts.
A newborn male is evaluated because of asymmetry of the orbit and eyelid. Physical examination shows a dozen 6- to 10-mm tan macules, axillary freckles, pigmented nodules on the iris, and soft, fleshy, raised, soft-tissue lesions on the forehead and scalp. Which of the following is the most likely diagnosis?
A) Goldenhar syndrome
B) Kabuki syndrome
C) Muenke syndrome
D) Neurofibromatosis
E) Nevus sebaceous
The correct response is Option D.
The patient described meets the criteria for neurofibromatosis 1 by clinical diagnosis. Genetic testing is also possible. To make a clinical diagnosis, two or more of the following criteria are required:
Six or more café au lait spots 5 mm or larger (prepubertal) or 15 mm or larger (post-pubertal)
Family history
Two or more neurofibromas
Freckling in armpits or groin
Lisch nodules
Bony dysplasia (sphenoid wing in this case)
Tumor on optic nerve
A patient with Goldenhar syndrome has facial asymmetry, but does not have café au lait spots. Colobomas are common in patients with this syndrome, but Lisch nodules are not.
Muenke syndrome is associated with craniosynostosis. Kabuki syndrome has distinctive associated facial features with long palpebral fissures, everted lower lids, broad nasal tip, arched brows, protruding ears, intellectual disability, and often microcephaly.
Nevus sebaceous is a well-circumscribed hamartomatous lesion composed of sebaceous glands. There is a risk for malignant transformation.
Which of the following craniofacial anomalies is associated with orbital hypotelorism?
(A) Crouzon syndrome
(B) Frontonasal dysplasia
(C) Frontonasal encephalocele
(D) Trigonocephaly
The correct response is Option D.
Hypotelorism, a decrease in intraorbital distance, occurs with trigonocephaly caused by metopic synostosis and holoprosencephaly. In patients with metopic synostosis, growth is inhibited perpendicular to the synostotic metopic suture, resulting in decreased interorbital distance as well as narrowing of the forehead. Holoprosencephaly results from a breakdown in the formation of prechordal mesoderm, leading to deficiencies of the structures located in the midline and narrowing of interorbital distance.
Hypertelorism, an increase in interorbital distance, is associated with numerous craniofacial disorders, including frontonasal encephalocele and median facial clefts occurring secondary to incomplete fusion of the medial orbital walls. Patients with more severe forms of craniosynostosis, such as Apert or Crouzon syndrome, may also have orbital hypertelorism. The hypertelorism seen in patients with frontonasal dysplasia occurs as a result of excess bone formation.
An 8-month-old female infant is brought to the office by her parents. Physical examination shows a wedge-shaped skull with a keel formation on the forehead, close-set eyes, and hollowness of the temporal fossa on both sides of the head. Premature cranial suture ossification at which of the following sites is the most likely cause of this patient’s condition?
A) Bicoronal
B) Lambdoid
C) Metopic
D) Sagittal
E) Unicoronal
The correct response is Option C.
Craniosynostosis refers to the premature fusion of one or more cranial sutures that make up the cranial vault and cranial base. Once this fusion occurs prematurely, the growth of the skull is altered and the development of the head takes on a characteristic morphologic shape that is determined by the fusing suture.
Trigonocephaly is classically characterized by a typically wedge-shaped skull when viewed from above; it originates from a premature stenosis of the metopic suture followed by a bilateral growth restriction of the forehead. This results in bitemporal narrowing and hypotelorism.
Plagiocephaly or unilateral coronal synostosis is characterized by the flattening of the forehead and frontoparietal region ipsilateral to the fused suture. As a result of this fusion, a compensatory bulge occurs in the opposite frontoparietal skull. The temporal fossa on the side of the fusion is convex and the ear becomes anteriorly displaced. The petrous portion of the temporal bone that contains the glenoid fossa is also displaced forward and the articulation with the mandible is displaced forward as a result. The nasal radix is also deviated toward the fused side and the tip of the nose is turned to the opposite side.
Ridging of the sagittal suture forms a narrow biparietal skull. Scaphocephaly shows compensatory growth in the frontal region or frontal bossing and/or occipital coning. There is associated enlargement of the head circumference. Sagittal synostosis remains the most frequent of the nonsyndromic craniosynostosis.
Unilateral lambdoid synostosis has ridging of the lambdoid suture, ipsilateral parieto-occipital flattening, prominence of the mastoid air cells, posterior displacement of the ear on the side of the occipital flattening, and scoliosis of the base of the skull, resulting in curvature of the cervical spine.
In patients with Treacher Collins syndrome, which of the following is a characteristic skeletal finding?
A) Brachycephaly
B) Hypertelorism
C) Macrogenia
D) Malar hypoplasia
E) Preaxial polysyndactyly
Correct answer is option D.
The characteristic skeletal finding in patients with Treacher Collins syndrome is hypoplasia of the malar bones, which often occurs in conjunction with clefting through the zygomatic arches. Patients also have hypoplasia of the maxilla and mandible and antegonial notching of the angle of the mandible. Occlusion is Angle class II; there is an anterior open bite and clockwise rotation of the occlusal plane. Effects on the temporomandibular joint are varied.
Brachycephaly, macrogenia, preaxial polysyndactyly, and hypertelorism do not occur in patients with Treacher Collins syndrome.
A 6-month-old girl is evaluated because of a “flat head.” Physical examination shows right posterior flatness of the cranium with a transcranial difference of 7 mm (minimal to moderate), and mastoid bossing on the ipsilateral side. The right ear and left forehead are anteriorly displaced. The parents report that the child prefers to turn her head to the right. The patient is otherwise healthy and reaching developmental milestones. Which of the following is the most appropriate next step in management?
A) Cranial orthotic molding helmet
B) Crib positioning and physical therapy only
C) CT scan of the head
D) X-ray studies of the skull
E) Reassurance
The correct response is Option C.
The clinical findings are suggestive of right unilateral lambdoidal synostosis (synostotic plagiocephaly). If craniosynostosis is suspected, the child may need surgical correction of the defect. Therefore, it is incorrect to pursue reassurance and conservative therapy without confirming the presence of a prematurely fused suture.
Lambdoidal synostosis is the least common premature fusion of a suture (2 to 3% of all synostosis cases). The physical findings have some similarities and distinct differences from the frequently seen positional deformational plagiocephaly. Positional plagiocephaly can have ipsilateral forehead protuberance and anterior displacement of the ipsilateral ear. Lambdoid synostosis, on the other hand, is associated with ipsilateral mastoid bossing and protuberance of the contralateral forehead. The position of the ear, once another landmark to help distinguish between positional and synostotic plagiocephaly, with lambdoid synostosis associated with posterior displacement of the ipsilateral ear, has been questioned in recent years. A literature review showed that the ipsilateral ear has been reported to be nondisplaced, anteriorly displaced, or posteriorly displaced. Therefore, the significance of the position of the ear is unclear at this time.
There is no need for plain film series before head CT scan. Imaging studies are performed to confirm synostosis before surgical intervention.
A 21-year-old man has an intercanthal distance of 37 mm (N = 30 mm), and an interorbital distance of 35 mm (N = 28 mm). Which of the following is the most likely diagnosis?
A ) Esotropia
B ) Exophthalmos
C ) Exotropia
D ) Hypertelorism
E ) Pseudohypertelorism
The correct response is Option D.
Orbital hypertelorism refers to a condition in which the interorbital distance, measured from dacryon (the junction of the frontal, lacrimal, and maxillary bones) to dacryon is increased. Orbital hypertelorism occurs in the setting of a number of conditions, including frontonasal malformations, craniofrontonasal dysplasia, craniofacial clefts, encephaloceles, and other mostly syndromic disorders. Some authors distinguish orbital hypertelorism from interorbital hypertelorism. In the latter, the distance between the medial orbital walls is increased just as in orbital hypertelorism but the lateral orbital walls are in a normal position. The mean interorbital distance is 28 mm in men and 25 mm in women.
Exotropia refers to a form of strabismus where the eyes are deviated outward (laterally), in contrast to esotropia, where the eyes are deviated inward (medially). Either exotropia or esotropia may exist in hypertelorism and data are not provided in the question to make this diagnosis. Pseudohypertelorism, or telecanthus, occurs when the intercanthal distance, defined as the distance between the medial canthal tendon insertions, is increased but the interorbital distance is normal. Telecanthus may give the appearance of hypertelorism, but can be differentiated on plain film x-rays or computed tomography. Exophthalmos refers to protrusion of the globe due to an increase in the size of the orbital contents in the presence of a normal bony orbit. Graves disease is one of the most common causes of exophthalmos. This is in contrast to exorbitism, which is protrusion of the globe due to a decrease in the capacity of the bony orbit, which may be seen in hypertelorism.
- Syndromic cleft palate is most commonly found in patients with which of the following syndromes?
- A ) Apert
- B ) Saethre-Chotzen
- C ) Stickler
- D ) Van der Woude
•Stickler syndrome accounts for 25% of syndromic cleft palate cases. As the most common form of syndromic cleft palate, this syndrome is also associated with ocular and auditory defects. Apert syndrome is an autosomal dominant form of syndromic craniosynostosis. Although this syndrome is defined by uni- or bicoronal craniosynostosis and acrosyndactyly, palatal clefting is not recognized as a syndromic component. Saethre-Chotzen syndrome is an autosomal acrocephalosyndactyly characterized by coronal craniosynostosis, low hairline, proptosis, antimongoloid slanting of palpebral fissures, and nasal deviation. Cleft palate defects are not associated with this syndrome. Van der Woude syndrome, associated with lower lip pitting and cleft palate, accounts for 1% of syndromic cleft defects.
•
A 6-month-old girl is brought to the office for evaluation of an isolated cleft of the soft palate. A photograph is shown. Which of the following genes is most likely defective in this patient?
A) FGFR2
B) IRF6
C) MSX1
D) SHH
E) TGF-beta
The correct response is Option B.
The patient has Van der Woude syndrome, an autosomal dominant condition affecting 1:35,000 to 1:100,000 persons. The pathognomonic features include lower lip pits, as seen in the photograph, and clefting of the palate, lip, or both. This syndrome is the most common single-gene cause of cleft lip and palate. IRF6 codes for a transcription factor that is involved in the early development. The mutated copy of the gene decreases the amount of active protein and results in the defects associated with this syndrome and popliteal pterygium syndrome. The remaining genes and their products have been implicated in craniofacial morphogenesis and etiopathogenesis of cleft lip/palate. Nevertheless, none are associated with lip pits or are an autosomal dominant cause of facial clefting.
For each patient or group of patients, select the most likely diagnosis (A-E).
(A) Apert syndrome
(B) Carpenter syndrome
(C) Crouzon syndrome
(D) Nager syndrome
(E) Pfeiffer syndrome
An infant has brachycephaly, hypertelorism, a bregmatic “bump,” and bony syndactyly of the fingers. A photograph is shown above.
Three members of a family have exorbitism, proptosis, retromaxillism, and recession of the frontal bone. There are no extremity abnormalities. A photograph is shown above.
The correct response for Item 49 is Option A and for Item 50 is Option C.
The infant has findings consistent with Apert syndrome, which is characterized by brachycephaly, hypertelorism with flattening of the face, strabismus, palsy of the ocular muscles, an antimongoloid slant of the palpebral fissures, and maxillary hypoplasia. Affected patients typically have a prominent bregmatic eminence or “bump.” There is bony syndactyly with complete fusion of the four fingers; the thumb is unaffected. Cutaneous syndactyly of the toes may be simple or complex. Although occurrence is typically sporadic, new mutations with autosomal dominant inheritance have been detected.
The affected family most likely has Crouzon syndrome, characterized by exorbitism, retromaxillism, inframaxillism, and paradoxical retrogenia. Inheritance is autosomal dominant, and occurrence is both sporadic and familial. Affected patients typically have recession of the frontal bone and supraorbital rim, midface retrusion, exorbitism with proptosis, and hypoplasia of the infraorbital rim. Hypertelorism, a bregmatic “bump,” and abnormalities of the hands are not characteristic.
Patients with Carpenter syndrome, or acrocephalopolysyndactyly, have craniosynostosis, shortened fingers, soft-tissue syndactyly, preaxial polydactyly, congenital cardiac disease, hypogenitalism, obesity, and umbilical herniation. The inheritance pattern of this syndrome is autosomal recessive.
Nager syndrome, or acrofacial dysostosis, is an autosomal recessive condition. Affected patients typically have short stature, cleft palate, and hypoplasia of the orbits, zygoma, maxilla, and mandible. Preaxial reduction defects occur in the upper, and sometimes lower, limbs. Hypoplasia or agenesis of the thumbs, radius, and one or more metacarpals is also characteristic.
Pfeiffer syndrome is an autosomal dominant disorder characterized by variable forms of craniosynostosis, acrocephalosyndactyly with broad thumbs and great toes, and severe midface hypoplasia.
A 9-year-old girl is brought for evaluation by her mother because of a 1-year history of headaches and precocious puberty. Physical examination shows café-au-lait spots over her abdomen and a bony prominence of her right parieto-occipital area. CT scan of the calvarium is shown. Which of the following is the most likely diagnosis?
A ) Klippel-Feil syndrome
B ) McCune-Albright syndrome
C ) Paget disease
D ) Proteus syndrome
E ) Renal osteodystrophy
The correct response is Option B.
The entity represented in the CT scan shown is fibrous dysplasia, which, in combination with precocious puberty and café-au-lait spots, is known as McCune-Albright syndrome. These patients typically have polyostotic fibrous dysplasia, most commonly affecting the skull, long bones, and ribs. Affected bone may show minor changes that are detectable only on imaging studies such as CT scans or bone scans. Such scans may also show severe change or overgrowth resulting in impingement of the optic nerves, mass effect on the brain, or disfigurement.
The precocious puberty in the scenario described is the result of gonadotropin-independent autonomous ovarian or testicular function and is more common in girls than in boys. Klippel-Feil syndrome is characterized by congenital fusion of any two of the seven cervical vertebrae. Traits of this condition include a short neck, low occipital hairline, and restricted mobility of the upper spine. Paget disease of the bone is also known as osteitis deformans and is a chronic disorder resulting in enlarged, deformed bones. It is typically diagnosed in the third or fourth decade of life by a blood test for alkaline phosphatase. The excessive breakdown and formation of bone can result in weakening of the bone.
Proteus syndrome causes atypical bone development and skin overgrowth. It is an extremely rare congenital disorder, and it is variable in its symptoms. Renal osteodystrophy is a bone mineralization deficiency resulting from electrolyte and endocrine abnormalities associated with chronic kidney disease.
A 10-year-old boy with a history of fronto-orbital advancement for craniosynostosis as an infant is brought to the office for evaluation. He would like to play football, but his parents are concerned about a persistent calvarial defect. Physical examination shows a 4 × 4-cm area of the right forehead that has no bony coverage; dural pulsations are observed. Which of the following is the most appropriate management?
A) Fabrication of a custom implant
B) Reconstruction with methylmethacrylate
C) Split calvarial bone grafting
D) Split rib grafting
E) Observation and delayed treatment until after completion of growth
The correct response is Option C.
This is a growing child with a large calvarial defect in a problematic location. Therefore, the defect needs to be addressed sooner rather than later, due to the potential risk for trauma and the child’s desire to be active. The gold standard for reconstruction of defects of this size and location is split calvarial bone grafting. Rib grafting would work, but would necessitate another donor site and contouring issues. A custom implant or methylmethacrylate would fix the problem in the short term, but would not grow with the child, necessitating additional surgery in the future.
An 18-month-old child is brought to the office after undergoing fronto-orbital advancement for metopic craniosynostosis. Which of the following is the earliest age the surgeon can order the x-ray studies and expect to be able to see frontal sinus development?
A) 1 year
B) 2 years
C) 4 years
D) 6 years
E) 10 years
The correct response is Option D.
Frontal sinus development is associated with specific age-related periods of growth of the skull. The frontal sinus is absent at birth and during the initial phase of growth of the skull. The sinus is visible only in x-ray studies at the end of the first period of skull growth. This is the time when the endocranial table of the skull ceases to grow and conforms to the general shape of the brain. This is not seen on x-ray studies until 6 years of age or 72 months.
A 20-year-old woman comes to the office to discuss reconstruction for hemifacial atrophy. A photograph is shown. The atrophy began when she was 13 years old and stabilized by 16 years of age. Workup suggests mixed connective tissue etiology that falls within the lupus spectrum. Which of the following is the most appropriate intervention to correct this patient’s deformity?
A) Buried anterolateral thigh free flap
B) Custom silicone prosthesis
C) Gracilis free flap
D) Injectable poly-L-lactic acid
E) Pedicled pectoralis major flap
The correct response is Option A.
Facial contour deformities can result from a variety of causes, can be unilateral or bilateral, and may range in severity. The most common cause is Romberg disease, progressive hemifacial atrophy. The cause is unknown but may be a localized form of scleroderma. Systemic lupus erythematosus and scleroderma are the most common connective tissue disorders resulting in soft-tissue atrophy with facial involvement.
Systemic lupus erythematosus is an inflammatory connective tissue disorder of unknown cause that predominantly affects women (90%). The clinical course can be variable and involve almost all organ systems. Cutaneous manifestations include soft-tissue atrophy, telangiectasia, and extremity ulcerations. Scleroderma refers to a spectrum of disorders characterized by diffuse fibrosis of the skin, blood vessels, synovia, and vital organs, such as the kidneys. Women are affected four times more frequently than men and the skin is involved in 95% of cases. The pathogenesis is unclear but may include vascular abnormalities (vasculitis), excess collagen deposition, and excess proteoglycan production in the involved tissues.
Surgical options for correction of hemifacial atrophy include alloplastic implants, free dermis/fat grafts, fat injections, and fasciocutaneous free tissue flaps. For mild defects, a custom alloplastic implant composed of silicone or porous polyethylene may be acceptable, but risks such as skin breakdown with implant exposure, scar contracture, and poor long-term patient satisfaction make an alloplastic option less desirable. In the patient described, the large nature of the defect and thin nature of her skin make an alloplastic option a poor choice.
Fat grafting using the techniques described by Coleman is a viable option for smaller defects. The large defect and need for multiple cycles of fat grafting make this a less desirable option in this patient. Various free flaps including omentum, parascapular, groin, and superficial inferior epigastric artery flaps have been described for correction of hemifacial atrophy. In this case, the anterolateral thigh flap is the most appropriate choice because it allows for harvest of a large amount of tissue, including skin, fat, fascia, and muscle.
A gracilis free flap is a great option for facial reanimation, but it would not have enough size and bulk to correct this patient’s deformity. Similarly, a pectoralis major flap would not be acceptable due to lack of bulk and inability to position the flap without tethering because of the pedicle. Additionally, it would be difficult to transpose the muscle high enough for full correction and an unaesthetic bulge would be notable in the neck.
A 4-month-old child has an abnormal head shape. Physical examination shows anterior displacement of the right ear and zygoma and the right side of the forehead; there is abnormal flattening of the right side of the occipital skull. The skull shape appears similar to a parallelogram. There is no bulging of the mastoid or ridging of the sutures.
These findings are most consistent with which of the following?
(A) Bilateral posterior deformational plagiocephaly
(B) Left-sided posterior deformational plagiocephaly
(C) Left-sided posterior lambdoidal craniosynostosis
(D) Right-sided posterior deformational plagiocephaly
(E) Right-sided posterior lambdoidal craniosynostosis
The correct response is Option D.
Plagiocephaly, or abnormal head shape, can be characterized as synostotic (resulting from craniosynostosis) or nonsynostotic (resulting from deformation or molding of the skull). Unilateral coronal and lambdoidal craniosynostosis are the most common causes of synostotic plagiocephaly. In contrast, deformational plagiocephaly, or skull molding, typically results from placing the infant in a fixed supine position for sleep. Posterior deformational plagiocephaly now occurs in one out of 70 infants. The increase in the incidence of this condition is due to recent recommendations by the American Academy of Pediatrics that infants be placed in the supine position during sleep to decrease the incidence of sudden infant death syndrome (SIDS).
Most infants with posterior deformational plagiocephaly have unilateral findings, including flattening of the occiput associated with anterior displacement of the ipsilateral ear. In severe forms of deformational plagiocephaly, there may be anterior displacement of the ipsilateral forehead and zygoma and widening of the ipsilateral palpebral fissure, resulting in a parallelogram-shaped cranium. If the condition is diagnosed early, treatment involves repeatedly repositioning the child out of the “flat spot.” However, cranial molding helmets may be required for those infants who have severe deformational plagiocephaly, who are resistant to repositioning, or in whom the condition is diagnosed late.
Infants with bilateral posterior deformational plagiocephaly have flattening of the occiput, bulging of the mastoid bilaterally, and bossing of the biparietal eminence. The suture is not ridged, and the ears are essentially symmetric.
Lambdoidal craniosynostosis is characterized by ipsilateral flattening of the occiput in combination with ridging of the fused lambdoid suture. There is compensatory bulging of the contralateral parietal skull and bulging of the ipsilateral mastoid skull, resulting in an inferior, not anterior, displacement of the ipsilateral ear. This gives the cranium a trapezoid-like shape. Appropriate management is craniofacial surgery with cranial vault remodeling at approximately 6 to 9 months of age.
A 5-year-old boy has an interrupted aortic arch, low-set ears, broad nasal bridge, velopharyngeal insufficiency, and hypoparathyroidism. A photograph is shown. Which of the following additional findings are most likely in this patient?
A) Facial asymmetry, microtia
B) High myopia, retinal detachment
C) Lower eyelid coloboma, downward-slanting palpebral fissures
D) Lower lip pits, salivary mucous drainage
E) T-cell immunodeficiency, chronic otitis media
The correct response is Option E.
22q11.2 Deletion syndrome (also known as DiGeorge, velocardiofacial, and conotruncal anomaly face syndromes) is often identified with help of the “CATCH 22” mnemonic, which refers to its characteristics of cleft palate, abnormal facies, thymic hypoplasia, cardiac malformations, and hypoparathyroidism. T-cell immunodeficiency and recurrent otitis media infections occur secondary to thymic hypoplasia and cleft-related ear anomalies.
Van der Woude syndrome is characterized by cleft palate and/or cleft lip, and lower lip pits/fistulae. Abnormal salivary gland morphology and hypodontia may also occur.
Treacher Collins syndrome (mandibulofacial dysostosis) is characterized by malar and mandibular hypoplasia, down-slanting palpebral fissures, lower eyelid coloboma, and ear anomalies.
Stickler syndrome type 1 is characterized by cleft palate, facies such as mid-face hypoplasia/retrusion, myopia with increased risk for retinal detachment, sensorineural hearing loss, and joint abnormalities including hypermobility and early-onset osteoarthritis.
Hemifacial microsomia is characterized by small and/or flattened maxillary, temporal, and zygomatic bones. Facial asymmetry is further apparent upon animation when soft tissue and nerve hypoplasia occur. Oral clefts and ear anomalies are commonly involved, as well as underdevelopment of the orbits.
Van der Woude syndrome, Treacher Collins syndrome, Stickler syndrome, and hemifacial microsomia are all unrelated to hypoparathyroidism.
A 3-year-old boy is brought for evaluation of a right frontotemporal cranial defect. A photograph is shown. Which of the following materials is most appropriate for reconstruction?
A) Autologous bone
B) Bone morphogenetic protein (BMP)
C) Calcium phosphate paste
D) Porous polyethylene
E) Titanium mesh
The correct response is Option A.
Autologous bone is the most appropriate material for inlay cranioplasty for a child this age. Each of the other options has been used in the pediatric population, but they carry important limitations. Porous polyethylene can be used to reconstruct large cranial defects, but is expensive to fabricate, has the potential to become unstable in a growing cranium, and has a higher overall infection rate than autologous bone. Bone morphogenetic protein (BMP) has shown promise in animal models for inducing bone formation in large cranial defects, but it is expensive for a defect of this size; it is a powerful mitogen and can result in dural ossification and, potentially, induction of tumor growth. Calcium phosphate pastes also have been advocated but are very expensive and for a defect of this size, the long-term potential osseointegration is dubious. Furthermore, the risk of infection and/or implant extrusion is higher. Titanium mesh may be appropriate for some children with limited life expectancy or functional needs, but these implants can be bent, offer limited impact resistance, and have a higher overall rate of complications than autologous bone.
A 6-year-old girl with 22q11.2 deletion syndrome is brought to the office by her parents. The parents report trouble understanding the child when she speaks. An incompetent velopharyngeal port is suspected. Which of the following is the most likely indication of this malfunction?
A) Consonant omission
B) Glottal stop
C) Hyponasality
D) Nasal air emission
E) Sibilant lateralization
The correct response is Option D.
Nasal air emission, along with hypernasality and facial grimacing, are all findings that are commonly observed in patients with an incompetent velopharyngeal port or velopharyngeal dysfunction (VPD). Consonant omissions and glottal stops are two common pathologic compensatory speech patterns that such patients exhibit but are not directly caused by VPD. Instead, they are maladaptive compensatory speech patterns often present in patients with VPD. Although some patients with VPD can have hyponasality or sibilant lateralization, these are unrelated to VPD or an attempt to compensate for VPD.
A 3-month-old male infant is brought to the office by his parents for evaluation of skull asymmetry that has worsened since birth. Birth history includes prolonged labor that required cesarean delivery. Physical examination shows flattening of the right posterior occiput with ipsilateral forehead bossing. From a superior view, the right ear is 1 cm more anterior than the left ear, and the anterior fontanelle is open without any bulging. The child’s head tilts to the right and has decreased range of motion when looking to the left. The left side of his neck feels tighter and more rigid compared with the right side. Which of the following is the most appropriate initial management of this child?
A) Cerebral palsy evaluation
A) Occupational therapy of the neck
A) Posterior cranial vault expansion
D) Passive molding helmet therapy
E) Sternocleidomastoid muscle release
The correct response is Option B.
This child demonstrates the classic presentation of deformation plagiocephaly with his posterior occiput flattening and compensatory ipsilateral forehead bossing with anterior shifting of his ear on the same side. There are many reasons for deformational plagiocephaly, especially with the current practice of “back to sleep.” Treatment of this focuses on removing the pressure on the affected side. His head tilt and decreased motion are consistent with torticollis. Initial treatment is stretching and occupational therapy to restore usage and balance of his neck muscles. Although helmet therapy can help alleviate pressure on this side, it is not addressing the issue. Further studies have demonstrated that deformational plagiocephaly can be treated with positional changes and behavior modification up until 7 to 8 months of age without difference in head asymmetry compared with those children who began helmet therapy at a younger age. There is no difference in children who fail positional changes compared with those who initiate helmet therapy immediately. At this child’s age of 3 months and with obvious torticollis, the most appropriate initial therapy should be focused on resolving his torticollis and giving him a trial of non-helmet therapy.
- An 8-year-old boy is brought to the office because of new-onset facial asymmetry that has slowly progressed over the past six months. The patient is otherwise healthy and has no other abnormalities. A current photograph is shown. Which of the following is the most likely diagnosis?
- A ) Binder syndrome
- B ) Craniofacial fibrous dysplasia
- C ) Oculoauriculovertebral spectrum
- D ) Romberg disease
- E ) Treacher Collins syndrome
- Romberg disease (progressive hemifacial atrophy, linear scleroderma, Parry-Romberg) is a progressive atrophy of one side of the face. Patients with Romberg disease often have a pigmented streak on the forehead, progressing to the”coupe de sabre” or facial depression. There is progressive atrophy of the skin, subcutaneous tissue, and bone. Many believe that it is a subset of localized or linear scleroderma.
- Patients with Binder syndrome, or nasomaxillary or premaxillary hypoplasia, have low-set and flat nasal tip, a short, retracted columella, Class III occlusion, and a columella and upper lip that is”drawn into” the floor of the nostrils.
- Fibrous dysplasia is a bony dysplasia, presenting as localized bony deformity (ground glass appearance on CT scan). Fibrous dysplasia may be limited to a localized area (monostotic) or may be generalized, affecting many sites (polyostotic), and is called McCune-Albright syndrome.
- Goldenhar syndrome (oculoauriculovertebral spectrum) is a subset of hemifacial microsomia and, in addition to microsomia of the face, includes vertebral anomalies and/or epibulbar dermoids.
- Treacher Collins syndrome is also known as mandibulofacial dysostosis. Patients with Treacher Collins have bilateral Tessier No. 6, 7, and 8 clefts and present with mandibular deformities, lower eyelid colobomas, auricular deformities, and hypoplastic zygomas.
•
Which of the following sutures (A-E) in the figures shown is the first to close in the nonpathologic state?
A ) Sagittal
B ) Metopic
C ) Coronal
D ) Lambdoid
E ) Squamosal
The correct response is Option B.
Physiologic or normal closure of the metopic suture occurs first. Recent studies report an earlier closure date than originally thought. The metopic suture may start to close as early as 6 months. The rest of the sutures identified close much later, with the sagittal suture (A) closing at 22 years, the coronal suture (C) closing at 24 years, the lambdoid suture (D) closing around 26 years, and the squamosal suture (E) closing at 35 years.
An otherwise healthy 10-month-old female infant is brought to the office because her parents are concerned about the appearance of her head. Examination shows left frontal and left posterior flattening with deviation of the nasal root to the left side. Which of the following is the most appropriate management?
A ) Molding helmet
B ) MRI of the head
C ) Positioning
D ) Surgery
E ) Observation only
The correct response is Option D.
In the scenario described, flattening of the left side of the forehead, deviation of the nasal root to the left side, and flattening of the left occiput are consistent with left coronal synostosis, and surgery is ultimately the treatment of choice. The scenario does not describe deformational plagiocephaly, which may be treated with a molding helmet, positioning, or observation. In an otherwise healthy baby with these findings, there is no indication for an MRI of the head.
The association between craniofacial defects and cardiac malformations in patients with velocardiofacial syndrome results from a disruption in the cellular development of which of the following?
A) Cardiogenic mesoderm
B) Ectodermal placodes
C) Lateral plate mesoderm
D) Neural crest
E) Somites
The correct response is Option D.
Neural crest cells derive from the ectoderm layer of the developing embryo, specifically the neuroectoderm of the forebrain, midbrain, and hindbrain. The neural crest contributes significantly to the craniofacial region, and also to the conotruncal endocardial cushions that are responsible for dividing the outflow tract of the heart into separate pulmonary and aortic components. Therefore, defects in neural crest cell development will frequently result in malformations of both the craniofacial area and cardiac septum. Examples of this association are: Treacher Collins syndrome, Pierre Robin sequence, 22q11.2 deletion syndrome, and oculoauriculovertebral syndrome.
The cardiogenic mesoderm derives from splanchnic (visceral) mesoderm, and contributes the precursor cells that differentiate into the endocardium and myocardium. It does not contribute to the head and neck.
Ectodermal placodes are separate from the neuroectoderm and consist of areas of thickened ectoderm that guide neural crest cells in forming the cranial sensory ganglia.
Lateral plate mesoderm is a derivative of the mesoderm layer of the embryo and contributes to the laryngeal cartilages and associated connective tissue of the head and neck.
Paraxial mesoderm produces somites that form the skull, meninges, voluntary craniofacial musculature, and dermis and connective tissue of the dorsal aspect of the head.
A 16-month-old male infant is brought to the physician because of congenital anomalies of both feet. The dorsal and plantar aspects are depicted in the photographs shown. Which of the following is the most likely diagnosis?
A) Apert syndrome
B) Crouzon syndrome
C) Jackson-Weiss syndrome
D) Pfeiffer syndrome
E) Saethre-Chotzen syndrome
The correct response is Option A.
Syndromic craniosynostosis often presents with findings in the hands and feet, sometimes referred to as acrocephalosyndactyly.
Apert syndrome is unique for having bilateral symmetric complex syndactylies involving nearly all digits, of both the hands and feet.
Crouzon syndrome usually has normal hands and feet.
Jackson-Weiss can have foot anomalies, such as short metatarsals.
Pfeiffer syndrome usually has broad thumbs and toes.
Saethre-Chotzen syndrome may have incomplete single syndactylies, but otherwise does not typically have extremity findings.
While performing a cranioplasty on a 6-year-old girl, the plastic surgeon accidentally drops the bone graft on the floor. Which of the following is the most appropriate next step?
A) Change the wound classification from 1 to 4
B) Decontaminate with triple antibiotic solution lavage and reuse the graft
C) Discard the graft and use an alloplastic material
D) Steam sterilize and reuse the graft
E) Wash with chlorhexidine and reuse the graft
The correct response is Option B.
In this circumstance, the surgeon should wash the graft with sterile triple antibiotic solution (0.1% gentamicin, 0.1% clindamycin, 0.05% polymyxin) and consider reusing it. This graft is not easily re-harvestable, and therefore reusing it is a reasonable alternative. Triple antibiotic solution is readily available and nontoxic to the bone graft, unlike iodine and chlorhexidine. Studies have shown that iodine does not have the antimicrobial effectiveness of other antimicrobials. Studies also show that while chlorhexidine is most effective in eradicating the microorganisms from the graft, its detrimental effect on corticocancellous bone prevents its use in this clinical scenario. If the dropped graft were skin, cartilage, or composite tissue, chlorhexidine would be the ideal antimicrobial.
Discarding the graft is not the ideal alternative given the extremely low likelihood of infection if reused after appropriate decontamination. In this clinical scenario, alloplastic materials are contraindicated due to the patient’s age. One should change the wound classification from 1 to 3, not 4. Steam sterilization is not used for such grafts in such an acute setting and this technique would potentially harm the graft.
A 6-year-old boy presents with a right-sided craniofacial cleft involving the medial third of the upper eyelid with canthal involvement and orbital malposition. Which of the following best represents the Tessier cleft classification for this patient?
A) No. 3
B) No. 5
C) No. 11
D) No. 13
The correct response is Option C.
The number 11 cleft is the cranial extension of the number 3 facial cleft. The cleft originates in the medial orbit and often involves the upper lid as a coloboma or a blepharon. The cleft may also extend into the eyebrow and up to the frontal hairline with a projection of hair extending into the medial third of the forehead.
The number 13 cleft is the paramedial extension of the number 1 facial cleft. The eyelid, eyebrow, and canthal structures are intact in the number 13 cleft, but the medial brow may be displaced inferiorly. There may be V-shaped paramedial projection of the frontal hairline as well as a frontal encephalocele. The skeletal abnormality of the cleft occurs between the nasal bone and the frontal process of the maxilla, causing hypertelorism and widening of the ethmoids, as well as frontal soft tissue and bony abnormalities.
The number 5 cleft begins medial to the lateral oral commissure, proceeding cephalad toward the middle third of the lower eyelid. There is frequently a soft tissue deficiency between the mouth and the lower lid, increasing the risk for corneal exposure. The skeletal defect of the cleft originates in the alveolus, lateral to the canine, and extends through the maxillary sinus to the inferolateral orbit, lateral to the infraorbital foramen.
The number 3 Tessier cleft is often referred to as the oro-naso-ocular cleft. When isolated to the lip, the number 3 cleft can mimic a common cleft lip, beginning at Cupid’s bow peak and extending to the lateral nasal sill. The cleft can extend through the lateral ala to the lower eyelid between the medial canthus and the inferior punctum. A significant soft tissue deficit of the midface may be present with inferior displacement of the lower lid and globe, as well as microphthalmia of the globe. The skeletal defect of the number 3 cleft originates in the alveolus, between the lateral incisor and canine, and extends cephalad to the lateral piriform aperture and medial orbit, terminating in the lacrimal groove of the inferior medial orbit.
A 12-year-old girl is evaluated for left-sided progressive hemifacial atrophy. Onset of symptoms started at 10 years of age. She has a history of migraines and experienced a seizure 1 year ago. Physical examination shows left-sided facial atrophy including cheek depression, enophthalmos, and dry skin with hyperpigmentation and alopecia. Maxillofacial CT scan shows associated skeletal changes. Which of the following clinical features is most likely associated with the early onset of the disease process?
A) Enophthalmos
B) Facial paralysis
C) Hyperpigmentation
D) Seizure disorder
E) Skeletal atrophy
The correct response is Option E.
Progressive hemifacial atrophy, also known as Parry-Romberg syndrome, is a rare disorder characterized by a self-limiting, slow, progressive, unilateral facial atrophy affecting the skin, subcutaneous tissue, muscle, and osteocartilaginous adjacent structures. It involves one or more trigeminal dermatomes with symptoms usually appearing in the first or second decade of life. The progression of atrophy usually lasts a variable 2 to 10 years. The etiology and pathogenesis of this disease are uncertain.
The severity of soft-tissue involvement appears to be independent of age of onset, facial distribution, or extent of the disease process; however, age of onset has been shown to correlate with the degree of bony hypoplasia. Onset of the disease at an earlier age during skeletal immaturity has a higher likelihood of skeletal involvement.
Clinical features of hemifacial atrophy include enophthalmos, cheek depression, deviation of the mouth and nose towards the affected side, ipsilateral show of teeth, and tongue atrophy. In addition, extracutaneous involvement has been described, including hemiatrophy of the contralateral or ipsilateral arm, trunk or leg, as well as dental, ocular, and neurologic abnormalities such as enophthalmos, headaches, facial pain, and seizures. An age correlation with these features has not been demonstrated. Facial paresthesia has been described but motor function remains intact.
A 1-month-old infant has right-sided microtia and hemifacial microsomia (shown). Weakness of which of the following branches of the facial nerve is demonstrated?
(A) Buccal
(B) Cervical
(C) Mandibular
(D) Temporal
(E) Zygomatic
The correct response is Option C.
The photograph of this patient demonstrates animation with symmetrical function of all branches of the facial nerve with the exception of the marginal mandibular branch on the right. The marginal mandibular branch is responsible for innervation of the lower lip depressor muscle, which is nonfunctional in this patient.
The second branchial cleft arch gives rise to the facial (VII) nerve. This nerve exits the skull at the stylomastoid foramen and contains motor and sensory fibers. It has six major branches: the temporal, zygomatic, buccal, mandibular, cervical, and auricular branches. The auricular branch separates before the facial nerve turns to the parotid gland. At the parotid gland, the facial nerve separates into two main divisions, the temporofacial and cervicofacial divisions, which further divide into the temporal, zygomatic, buccal, mandibular, and cervical branches.
Hemifacial microsomia produces anomalies in skeletal, soft tissue, and neuromuscular structures derived from the first and second branchial arches. It is the second most common congenital facial anomaly and is unilateral in 80% of affected patients. Usually, patients have some form of microtia and hypoplasia of the mandibular ramus. They may also have macrostomia, facial paralysis, and hypoplasia of the muscles of mastication.
A 5-year-old boy is referred for evaluation of facial asymmetry. His parents report that they have noticed asymmetry since birth and believed it would improve with time, but it has not. The patient has no history of surgery or physical therapy. On physical examination, the physician notes skull asymmetry and facial asymmetry, including the orbits; there is no obvious neck tilt. A CT scan of the head is most likely to show which of the following?
A) Deformational plagiocephaly
B) Lambdoid craniosynostosis
C) Torticollis
D) Unicoronal craniosynostosis
E) Zygomaticotemporal craniosynostosis
The correct response is Option A.
Since the early 1990’s when positional deformational cranial deformities were first identified with the “back-to-sleep” campaigns, they have now become the most common etiology of cranial and facial asymmetry. Prior to that time, torticollis was the most common cause. Craniosynostosis is a much less frequent cause and occurs in the following order of decreasing frequency: unicoronal, lambdoid, zygomaticotemporal. Correct identification of most of the causes of this clinical constellation of findings can be made on clinical examination, but most would still get a CT scan to verify the diagnosis and rule out associated anomalies except for torticollis, which is always a clinical diagnosis.
A 6-month-old boy is referred by a pediatrician for evaluation of asymmetric shape of the skull. Physical examination shows flattening of the right posterior skull, prominence of the right side of the forehead and zygomatic complex, and forward advancement of the right ear. Which of the following is NOT an option in the management of this patient €™s condition?
(A) Observation
(B) Frequent repositioning exercises
(C) Helmet-molding therapy
(D) Three-dimensional CT scan of the head
(E) Surgical remodeling of the cranial vault
The correct response is Option E.
Surgical management of plagiocephaly is not indicated at this young age.
Since the €œback €‘to €‘sleep € campaign began recommending that young children be placed on their backs to sleep in an effort to limit the incidence of sudden infant death syndrome (SIDS), many children have been referred for evaluation of plagiocephaly. Physical examination continues to be the diagnostic method for separating plagiocephaly as a result of positional molding (ie, external pressures) from synostotic plagiocephaly.
Positional plagiocephaly results from external pressure on the skull €”in this case, the right posterior skull. As the brain grows, it pushes the ipsilateral structures forward, including the zygomatic complex, forehead, and ear. From a vertex view, the skull takes on the appearance of a parallelogram.
In contrast, when a growth suture is fused as in craniosynostosis, growth is restricted perpendicular to the suture and accelerated parallel to that suture (Virchow €™s Law). In cases of suspected lambdoid synostosis, the growth of the contralateral forehead is accelerated and the contralateral structures (such as the ear) are advanced. From the vertex, the skull becomes trapezoid in appearance. Similarly from a posterior view, there is ipsilateral mastoid bossing and the tilt.
A CT scan may be obtained for many reasons but is not necessary to establish the diagnosis in this patient. Physical examination should be adequate to diagnose positional plagiocephaly.
Treatment of positional plagiocephaly initially involves behavioral changes and frequent repositioning of the infant. Prone playtime activities are encouraged. Sometimes physical therapy is required. Custom helmet molding therapy is recommended for severe skull asymmetry or for patients with other developmental issues that would complicate behavioral modifications or repositioning techniques. In this child, poor muscle tone and slight developmental delay lead us to recommend helmet therapy. It is not unusual for the posterior fontanelle to be closed at this time.
A 6-year-old boy is brought to the office by his parents for consultation regarding bilateral congenital facial palsy and syndactyly of the hands. He underwent surgical correction of strabismus five years ago. Which of the following conditions is the most likely cause of this patient’s symptoms?
A) Apert syndrome
B) Bell palsy
C) Goldenhar syndrome
D) Hemifacial microsomia
E) Möbius syndrome
Correct answer is option E.
Möbius syndrome is a developmental disorder characterized by bilateral facial palsy and abducens nerve paralysis. Strabismus surgery is performed to correct paralysis of lateral gaze. Limb abnormalities, including clubfeet, syndactyly, and rudimentary fingers or toes, have been reported in 25% of cases. Additional cranial nerves (III, V, IX, XI, and XII) may be involved in Möbius syndrome, and some patients may present with congenital unilateral or partial facial paralysis. Hemifacial microsomia is a morphogenetic anomaly that can affect the skeletal, soft tissue, and neuromuscular structures derived from the first and second branchial arches. Typical cases have hypoplasia of the mandible that may be accompanied by hypoplasia of the zygoma and maxilla. Because the facial nerve is derived from the second branchial arch, patients with hemifacial microsomia can present with a congenital facial palsy. Goldenhar syndrome is hemifacial microsomia with epibulbar dermoids and vertebral anomalies. Apert syndrome is characterized by coronal craniosynostosis, syndactyly, and retardation. Bell palsy is a demyelinating inflammatory process of the facial nerve that classically presents as an acute unilateral facial paralysis and is believed to be caused by the herpes simplex virus.
Which of the following substances has been shown to be associated with the mechanisms of cranial suture fusion in animal models?
(A) Epidermal growth factor (EGF)
(B) Interleukin-6 (IL-6)
(C) Prostaglandin-E2 (PGE2)
(D) Transforming growth factor-beta (TGF-B)
(E) Tumor necrosis factor-alpha (TNF-B)
The correct response is Option D.
Although the mechanisms of action resulting in premature fusion of cranial sutures, or craniosynostosis, are unknown, experimental animal studies have shown that transforming growth factor-beta (TGF-B) plays a role in the fusion of posterior frontal sutures. According to immunolocalization techniques, certain isoforms of TGF-B are expressed during fusion of posterior frontal sutures in rat and mouse models; increased immunoreactivity of isoforms of both TGF-B and insulin-like growth factor (IGF) has also been shown to occur during premature fusion of the sagittal, coronal, and lambdoid sutures in humans. Qualitative analysis of TGF-B protein levels in organ culture models has also demonstrated increased levels of TGF-B during the period of active suture fusion.
Although basic fibroblast growth factor (b-FGF) has also been implicated in the process of cranial suture fusion, epidermal growth factor (EGF) has not been shown to be associated with suture fusion. Interleukin-6 (IL-6), prostaglandin-E2 (PGE2), and tumor necrosis factor-alpha (TNF-B) have been shown to mediate inflammatory responses but not to affect cranial suture fusion.
A 4-month-old male infant is brought to the office for evaluation of an abnormal head shape that has been present since birth. The patient is developmentally appropriate and has no other abnormalites. A photograph is shown. At which of the following ages does the involved suture normally fuse?
A) 2 weeks
B) 8 months
C) 3 years
D) 10 years
E) 22 years
The correct response is Option B.
The patient in the photograph has metopic craniosynostosis. The primary clinical features of trigonocephaly are a palpable metopic ridge, bossing with thickened bone at the glabella, bifrontal narrowing, recession of the superior orbital rims, temporal narrowing, and hypotelorism. Trigonocephaly is also known as a keel-shaped deformity.
The metopic suture is the first cranial suture to fuse and this usually occurs at approximately 8 months of age. The other cranial sutures generally fuse in adulthood.
Treatment depends on the age of presentation and extent of deformity. Options for management are varied and range from no intervention for minimal deformity, burring of the metopic ridge only, endoscopic synostectomy with helmet therapy, and bilateral fronto-orbital advancement.
Which of the following best describes the growth pattern of the calvarium relative to a fused suture?
A) Oblique
B) Parallel
C) Perpendicular
D) No relation
The correct response is Option B.
Virchow’s law states that bone surrounding a prematurely fused suture only grows parallel to the suture, while growth perpendicular to the suture is restricted. This contributes to the characteristic patterns of cranial shape in the various types of synostoses. Sagittal fusion yields a skull that is long in the anteroposterior direction and narrow bitemporally. Metopic fusion yields a triangular-shaped skull due to constriction caused by the anterior, medial metopic suture. Unilateral coronal synostosis yields an asymmetric skull, notably for a trapezoidal appearance from the overhead inspection. While there may be some ridging above and below the affected suture, this does not represent the growth of normal calvarial bone.
A 3-month-old male infant is evaluated because of bilateral preauricular pits and small ears. Physical examination is otherwise unremarkable. The patient failed the newborn hearing test; the parents have not repeated the test. They are not very concerned, because the father is also hearing impaired. Which of the following is the most appropriate next step in management?
A) Echocardiography
B) MRI of the brain
C) Renal ultrasound
D) Repeat hearing test
E) Surgical treatment of the preauricular pits
The correct response is Option C.
This child most likely has branchio-oto-renal (BOR) syndrome, an autosomal dominant syndrome. Two genetic mutations (EYA1 and SIX1) have been identified. The clinical findings include auricular malformations, preauricular skin pits, hearing loss, branchial fistulae, external auditory canal stenosis, and renal anomalies. Renal ultrasound should be performed to rule out renal anomalies. Family history of hearing loss should also raise suspicion.
Ear anomalies can also be associated with the oculo-auriculo-vertebral spectrum (OAV), within which hemifacial microsomia falls. OAV spectrum may have associated mandibular, facial nerve, and ocular findings (e.g., epibulbar dermoid, coloboma). Other possible associations include congenital heart defects, such as ventricular septal defect or tetralogy of Fallot. This child does not have other craniofacial findings of hemifacial microsomia, thus cardiac echocardiography is not indicated.
Internal cerebral arterial anomalies may be associated with PHACE syndrome (P, posterior fossa; H, hemangioma; A, arterial anomalies; C, cardiac defects; E, eye anomalies). Investigation of cerebral circulation by MRI of the brain would be prompted by a facial hemangioma, which is absent in this child.
While resection of the preauricular pits may be offered, it does not aid in ruling out other findings and there is no urgency to perform surgical resections in this child at 3 months of age.
Repeating a hearing test will not confirm or rule out OAV or BOR syndrome.
In the Tessier system, which of the following classifications represents the most common facial cleft?
(A) Tessier No. 0
(B) Tessier No. 3
(C) Tessier No. 7
(D) Tessier No. 9
(E) Tessier No. 14
The correct response is Option C.
The most common facial cleft is a Tessier No. 7. This is a cleft that begins at the lateral oral commissure and extends laterally. From a soft-tissue standpoint, it creates macrostomia. A Tessier No. 0 cleft involves the midline of the upper lip and nose. The Tessier No. 14 cleft is the cranial extension of this. A Tessier No. 3 cleft involves the lateral nasal ala and the medial canthus of the eye. A Tessier No. 9 cleft is actually the least common cleft. It extends from the superolateral orbit into the temporal region.
Mutations in the genetic loci for the fibroblast growth factor receptors (FGFR) have been shown to occur with greater frequency in patients with synostosis of which of the following sutures?
(A) Coronal
(B) Lambdoid
(C) Metopic
(D) Sagittal
(E) Squamosal
The correct response is Option A.
Mutations in the genetic locus for fibroblast growth factor receptor 3 (FGFR-3) have been found, in a greater than expected incidence, in both unilateral and bilateral coronal synostoses. Although these craniofacial malformations are thought to occur sporadically, this genetic link may explain the familial occurrence of coronal synostosis in some patients.
According to the results of recent studies, the craniofacial dysmorphology was more severe in patients who had the FGFR-3 mutation than in patients who did not carry the mutation, and outcomes of reconstructive surgery were poorer.
A 9-month-old boy is brought to the office because of a midline glabellar mass. The parents report that it has enlarged gradually since they first noticed it 6 months ago; it does not change in size when the patient cries. Physical examination shows a nonmobile, firm, and nontender mass. The nasal root is not broadened, and intercanthal distance is within normal limits. Which of the following is the most likely diagnosis?
A ) Dermoid cyst
B ) Encephalocele
C ) Glioma
D ) Hemangioma
E ) Pilomatricoma
The correct response is Option A.
The most likely diagnosis in the scenario described is a dermoid cyst. Nasal dermoids are the most common congenital nasal mass. Dermoid cysts most often occur in children in the lateral brow or midline glabellar region. They generally grow slowly, and intracranial communication should be ruled out with either a CT scan or MRI. Intracranial communication is less likely in this scenario because there is no broadening of the nasal root or increased intercanthal distance.
An encephalocele is a midline malformation that is present at birth and addressed shortly thereafter. On physical examination, it would be soft and mobile. CT scan or MRI would confirm this diagnosis. A hemangioma or encephalocele may change size with crying.
A glioma is heterotopic neural tissue left during the regression of neurologic tissue during embryonic development. Like an encephalocele, broadening of the nasal root and widened intercanthal distance are common.
A hemangioma typically presents with sporadic growth during the first 12 months of life, then it reaches a plateau and eventually regresses in most cases. On physical examination, this would be neither fixed nor firm and is typically discolored.
A pilomatricoma is a rare, benign, circumscribed epithelial neoplasm that is derived from hair matrix cells. It is classically not fixed and very superficial.
A 5-year-old girl has craniosynostosis, a low-set hairline, ptosis, and 4th/5th-digit syndactyly of both hands. Genetic testing is most likely to show a mutation in which of the following genes?
A) EFNB1
B) FGFR2
C) FGFR3
D) MPDZ1
E) TWIST1
The correct response is Option E.
The clinical picture is consistent with Saethre-Chotzen syndrome. It is an autosomal dominant condition defined by a genetic mutation or deletion affecting the TWIST1 gene or chromosome 7p21. FGFR2 mutations are predominantly associated with Apert, Crouzon, and Pfeiffer syndromes. FGFR3 mutations are associated with Muenke syndrome and Crouzon syndrome with acanthosis nigricans. EFNB1 is associated with craniofrontonasal syndrome. MPDZ1 is associated with hydrocephalus.
A 7-year-old girl is brought to the office for consultation regarding the congenital malformation shown. Which of the following anatomic structures is absent in this patient?
(A) Anterior nasal spine
(B) Dentoalveolus
(C) Hard palate
(D) Maxilla
(E) Nasal bones
The correct response is Option A.
The anterior nasal spine is absent in patients with Binder syndrome. In 1962, Von Binder described a form of nasomaxillary hypoplasia that included nasomaxillary hypoplasia, a convex lip, a vertically short nose, a flat frontonasal angle, an absent anterior nasal spine, limited nasal mucosa, and hypoplastic frontal sinuses. It is believed to be caused by a disturbance of the prosencephalic induction center at a critical phase in development.
Which of the following statements is correct about Tessier clefts No. 3, No. 4, and No. 5?
A) Tessier No. 3 involves the alveolar ridge, while Tessier No. 5 does not
B) Tessier No. 3 is medial to the infraorbital nerve, while Tessier No. 4 is lateral
C) Tessier No. 3 only affects the oral region, while Tessier No. 4 only affects the orbital region
D) Tessier No. 4 involves the piriform aperture, while Tessier No. 5 does not
E) Tessier No. 4 is medial to the infraorbital nerve, while Tessier No. 5 is lateral
The correct response is Option E.
Tessier No. 3 and No. 4 are medial to the infraorbital nerve, but Tessier No. 5 is lateral.
Tessier No. 3 involves clefts of the nose, orbit, and lip (naso-oral-ocular cleft), whereas Tessier No. 4 involves the lip and orbit (oral-ocular cleft), and the nose is uninvolved.
Tessier No. 5 involves oral, cheek (maxillary sinus), and orbital cleft and is the rarest.
A 3-year-old girl with craniofrontonasal dysplasia presents to the craniofacial team because of moderate exorbitism, severe central sleep apnea, and severe hypertelorism (interocular distance of 40 mm). Which of the following is the most appropriate management plan for this patient?
A) Continuous positive airway pressure until 6 or 8 years of age
B) Facial bipartition surgery within the next year
C) Immediate tracheostomy and assisted ventilation
D) Monobloc advancement surgery within the next year
E) Orbital box osteotomy surgery within the next year
The correct response is Option C.
Tracheostomy and assisted ventilation is the most appropriate management. The patient described demonstrates a significant number of central and mixed apneas, indicating that the initiation of breathing by central drive is diminished and that tracheostomy with assisted ventilation set at a base ventilation rate, such as synchronized intermittent mandatory ventilation (SIMV), would be optimal to ensure adequate minute ventilation.
Continuous positive airway pressure (CPAP) until 6 to 8 years of age and then a facial advancement is incorrect. CPAP will assist with obstructive apnea, but will not treat central apnea because no baseline ventilation rate is set using CPAP mode.
Hypertelorism surgery within the next year is incorrect because hypertelorism surgery will not treat sleep apnea without facial advancement. Only the wide orbit position would be treated with an orbital box osteotomy or facial bipartition without advancement.
Facial advancement surgery within the next year is incorrect because advancement will correct the obstructive apnea and a portion of the mixed apneas, but will not address the central sleep apnea. Observation with tracheotomy and assisted ventilation will be more prudent until the central apnea rate improves, usually after time with maturation. The patient described will still have persistent central apnea and is at higher risk for relapse of the face due to the age of under 6 years.
A 10-month-old male infant has undergone a cranial vault remodeling for sagittal synostosis. The child has no other comorbidities. On the second postoperative day, the patient’s serum sodium concentration decreases from 140 to 129 mEq/L. Laboratory studies show a suppressed plasma aldosterone concentration. The suspected diagnosis is cerebral salt wasting (CSW). An increase in which of the following is most likely to support this diagnosis?
A) Plasma antidiuretic hormone concentration
B) Plasma sodium concentration
C) Plasma volume
D) Serum uric acid concentration
E) Urine output
The correct response is Option E.
Cerebral salt wasting (CSW) is a condition that combines renal sodium and fluid loss after intracranial injury with an overall negative sodium balance and volume contraction. It responds to fluid and salt replacement. It contrasts with the syndrome of inappropriate antidiuretic hormone (SIADH), which is part of the differential diagnosis in hyponatremia as follows:
Mutation of the fibroblast growth factor receptor (FGFR) has been most commonly associated with which of the following single-suture synostoses?
(A) Lambdoid
(B) Metopic
(C) Sagittal
(D) Squamosal
(E) Unicoronal
The correct response is Option E.
Mutation of FGFR3, located at chromosome 4p16, has been found to cause unicoronal synostosis. This suggests a genetic basis for certain forms of synostotic frontal plagiocephaly. FGFRs regulate cell growth and bony proliferation. Mutations in FGFRs have been associated with syndromic craniosynostoses, such as in Pfeiffer, Crouzon, and Jackson-Weiss syndromes.
Evaluation of patients with unicoronal synostosis showed more severe cranial dysmorphology and a higher number of surgical revisions in those with FGFR3 mutation for facial dysmorphology compared with those without the mutation. This finding has led to genetic screening for all patients with unicoronal synostosis to better counsel patients and anticipate surgical outcomes.
Which of the following findings is commonly caused by non-syndromic unicoronal synostosis?
A) Anterior displacement of the ipsilateral ear
B) Deviation of the root of the nose to the contralateral side
C) Flattening of the ipsilateral aspect of the occiput
D) Occlusal cant up on the ipsilateral side
E) Recession of the contralateral forehead
Correct answer is option A.
Nonsyndromic unicoronal synostosis commonly causes anterior displacement of the ipsilateral ear toward the affected suture. It also typically results in ipsilateral flattening and contralateral bossing of the forehead and deviation of the root of the nose to the ipsilateral side toward the affected suture. Unicoronal synostosis does not typically affect occlusion on either side and is not likely to cause significant change in occipital shape.
In patients with Binder syndrome, the most likely physical finding is hypoplasia of which of the following structures?
(A) Anterior cranial base
(B) Anterior nasal floor
(C) Anterior wall of the maxilla
(D) Medial orbital wall
(E) Nasal septum
The correct response is Option B.
A patient with Binder syndrome, or maxillonasal dysplasia, typically has a shortened nose with flattening of the nasal bridge and perialar regions. The columella is shortened, the nasolabial angle is acute, and the upper lip is convex. The anterior nasal spine and frontonasal angle are absent. Occlusion is Angle class III. Binder syndrome is caused by hypoplasia of the anterior nasal floor (fossa praenasalis) and localized symmetric maxillary hypoplasia in the region of the alar rims. Nasal views show a retracted columellar-lip junction, a perpendicular alar-cheek junction, a convex upper nasal tip with a wide, shallow philtrum, crescent-shaped nostrils without a sill, a low-set and flat nasal tip, and a stretched and shallowed cupid’s bow. The triangular flair typically seen at the base of the nose is instead absent.
In patients with Binder syndrome, the primary goal of surgery is increasing the length of the nose and the projection of the nasal tip. This can be achieved by performing Le Fort I osteotomy, Le Fort II osteotomy, or a combination of both procedures, as well as compensatory orthodontic treatment. Autogenous bone and cartilage grafts may be required to reconstruct the nose.
Which of the following congenital clefts involves the lower eyelid?
A) 0
B) 1
C) 5
D) 8
E) 9
The correct response is Option C.
Craniofacial clefts are highly variable and can range from mild forme fruste to severe with involvement of all the layers of the soft tissue and skeletal structures. Tessier described a classification system still in use today based on the bony and soft-tissue landmarks involved in the clefts. Corrective surgery is required in stages, with the timing based on the level of functional problems, including ocular exposure, and airway problems early on. The clefts can be broadly grouped into midline and paramedian clefts (numbers 0 to 14, 1 to 13, 2 to 12), oro-nasal-ocular clefts (numbers 3 to 11, 4 to 10, 5 to 9), and lateral clefts (numbers 6, 7, and 8). In this system, the numbers 0 to 6 refer to clefts below the orbital and numbers 8 to 14 are above the orbit, with 7 being truly lateral.
A 3-year-old girl with mid face hypoplasia, proptosis, and malocclusion is brought to the office for consultation. Which of the following is the most likely occlusal relationship in this patient?
- A ) Angle class I
- B ) Angle class II, division 1
- C ) Angle class II, division 2
- D ) Angle class III
•The patient described with Crouzon disease and a hypoplastic mid face has an Angle class III malocclusion. Angle class I is characterized by the alignment of the mesiobuccal cusp of the maxillary first molar with the mesiobuccal groove of the mandibular first molar. Angle class II is characterized by the alignment of the buccal groove of the lower first molar distal to the mesiobuccal cusp of the upper first molar. Angle class II division 1 is associated with flaring of the maxillary incisors and increased overjet. Angle class II division 2 is associated with lingually displaced maxillary teeth and excessive labial inclination of the maxillary central incisors. Angle class III is characterized by the alignment of the buccal groove of the lower first molar mesial (anterior) to the mesiobuccal cusp of the upper first molar.
A 16-year-old girl is brought to the office for consultation regarding reconstruction to correct hemifacial atrophy. The parents first noticed the condition when the patient was 6 years old; it has been stable for 18 months. Physical examination shows an asymmetric face with atrophy of the right side. There is significant unilateral atrophy of skin, subcutaneous tissue, and bone. Facial reconstruction is planned. Which of the following is the most appropriate method of reconstruction?
A) Latissimus dorsi free flap
B) Omental free flap
C) Osteocutaneous fibula flap
D) Parascapular free flap
E) Silicone injection
The correct response is Option D.
Of the choices given, the most appropriate method of reconstruction in the patient described, who has Romberg disease, is a parascapular free flap.
Patients with Romberg disease may have deformities composed of skin, subcutaneous tissue, muscle, and bone. Options for reconstruction include synthetic material and implants, biologic fillers, bone grafts, fat grafts, and free tissue transfer.
Facial contouring with silicone injection has been unsatisfactory with severe scarring, contracture formation, and skin breakdown. Removal of liquid silicone can be a very challenging task and can make future reconstructions more difficult. Muscle and myocutaneous flaps are not ideal, as they can be too bulky, and the eventual muscle atrophy leads to unpredictable long-term results. Omental flaps for facial recontouring have been described, but they have several drawbacks. These include the need for an intra-abdominal harvest and the difficulty in long-term flap fixation, with eventual descent given the absence of dermal or fascial components to be used in fixation. The parascapular flap provides a versatile source of composite tissue that remains relatively stable as it matures, and it provides tissue components for appropriate fixation.
A 2-month-old infant has a facial cleft extending from the upper lip through the nasal ala and into the medial canthal region. Which of the following is the most appropriate Tessier classification of this cleft?
(A) No. 1
(B) No. 2
(C) No. 3
(D) No. 4
(E) No. 5
The correct response is Option C.
As shown below, the most appropriate classification for this cleft is Tessier number 3 because this is the only classification in which the cleft involves the nasal ala and medial canthus. A cleft classified as number 0 involves the midline of the nose. A cleft classified as number 1 or 2 involves the nasal ala but is medial to the eye. A cleft classified as number 4 or 5 is lateral to the nose and typically involves the lower eyelid.
Which of the following craniofacial anomalies does NOT demonstrate genetic transmission?
(A) Acrocephalosyndactyly
(B) Apert syndrome
(C) Craniofacial microsomia
(D) Mandibulofacial dysostosis
(E) Nager syndrome
The correct response is Option C.
Craniofacial microsomia is the most common major craniofacial anomaly and does not demonstrate genetic transmission. Instead, it is believed to be caused by an intrauterine event, such as occlusion of the stapedial artery or development of hematoma. Craniofacial microsomia is far more likely to be unilateral (hemifacial) than bilateral. It manifests as facial paralysis and malformations of the mandibular ramus. Growth of the mandibular condyle is impaired, resulting in inadequate development of the mandible and the craniofacial osseous complex on the affected side.
Both Apert and Crouzon syndromes, which are syndromes of acrocephalosyndactyly, are inherited as autosomal dominant conditions. Their genetic mutations have been linked to fibroblast growth factor receptor-2 (FGFR-2). These syndromes are characterized by craniosynostosis, exorbitism, and midface retrusion; however, each is differentiated by its associated extremity findings. Patients with Apert syndrome have severe syndactyly of the middle three digits of the hands and feet, often with a common nail. In Crouzon syndrome, the extremities are normal.
Mandibulofacial dysostosis, also known as Treacher Collins syndrome, is an autosomal dominant disorder; its gene has been mapped to chromosome 5q31.3-q33.3. Patients with this condition have facial clefting, an antimongoloid slant to the palpebral fissures, colobomas of the lower eyelids, absence of eyelashes on the medial portion of the lower eyelid, preauricular displacement of hair, malar and mandibular defects, and micrognathia.
Nager syndrome is an autosomal recessive disorder that has facial characteristics similar to Treacher Collins syndrome. Affected patients also have hypoplasia of the thumbs, metacarpals, and radius.
According to Tessier’s classification, patients with which of the following types of clefts have displacement of the medial canthus of the eyelid?
(A) No. 1
(B) No. 3
(C) No. 5
(D) No. 7
(E) No. 9
The correct response is Option B.
The Tessier classification of craniofacial clefting, which was first proposed in 1973, integrates both tissue findings and underlying skeletal deformities. According to the Tessier system, displacement of the medial canthus of the eyelid is characteristic of the No. 3 cleft. This cleft begins in the alveolus, between the lateral incisor and canine, and extends through the maxilla and into the lacrimal bone. It is often referred to as a “naso-ocular” cleft because the inferomedial wall of the orbit is absent. Other associated soft-tissue deformities include shortening of the nose, colobomas of the nasal alae and the lower eyelids medial to the punctum, obstruction of the nasolacrimal duct, and malformations of the lower canaliculus. The lip deformity is similar to cleft lip.
The Tessier No. 1 cleft lies just lateral to the midline, beginning at the cupid’s bow and passing through the dome of the nostril lateral to the anterior nasal spine. Notching of the alar dome is a distinctive feature. The nasal bone may be absent, but the septum is unaffected. Hypertelorism and encephalocele may also be associated.
The No. 2 cleft is exceedingly rare and may be a transition between the No. 1 and No. 3 clefts.
The Tessier No. 4 cleft passes between the piriform aperture and the infraorbital foramen. It begins lateral to cupid’s bow and the philtrum and then passes lateral to the nasal ala and onto the cheek, terminating in the lower eyelid medial to the punctum. The medial canthal tendon is unaffected.
The rare Tessier No. 5 cleft begins behind the canine and extends through the maxillary sinus to the orbital floor. Colobomas of the lateral lower eyelids and clefting of the upper lip medial to the oral commissure are associated.
The No. 6 cleft is the incomplete form of Treacher Collins syndrome. This cleft passes inferior and lateral to the oral commissure, toward the angle of the mandible. Colobomas of the lateral lower eyelids occur.
The Tessier No. 7 cleft is the most common atypical cleft; it occurs in one of every 3000 births. It has sporadic transmission and highly variable expressivity. The zygomatic arch is typically absent.
The No. 8 cleft extends from the lateral commissure of the palpebral fissure to the temporal region and is also associated with colobomas of the lower eyelids.
The Tessier No. 9 cleft is merely a supraorbital extension of the No. 5 cleft.
A 6-year-old boy is brought to the office by his parents for consultation regarding bilateral congenital facial palsy and syndactyly of the hands. He underwent surgical correction of strabismus five years ago. Which of the following conditions is the most likely cause of this patient=s symptoms?
(A) Apert syndrome
(B) Bell palsy
(C) Goldenhar syndrome
(D) Hemifacial microsomia
(E) Möbius syndrome
The correct response is Option E.
Möbius syndrome is a developmental disorder characterized by bilateral facial palsy and abducens nerve paralysis. Strabismus surgery is performed to correct paralysis of lateral gaze. Limb abnormalities, including clubfeet, syndactyly, and rudimentary fingers or toes, have been reported in 25% of cases. Additional cranial nerves (III, V, IX, XI, and XII) may be involved in Möbius syndrome, and some patients may present with congenital unilateral or partial facial paralysis. Hemifacial microsomia is a morphogenetic anomaly that can affect the skeletal, soft tissue, and neuromuscular structures derived from the first and second branchial arches. Typical cases have hypoplasia of the mandible that may be accompanied by hypoplasia of the zygoma and maxilla. Because the facial nerve is derived from the second branchial arch, patients with hemifacial microsomia can present with a congenital facial palsy. Goldenhar syndrome is hemifacial microsomia with epibulbar dermoids and vertebral anomalies. Apert syndrome is characterized by coronal craniosynostosis, syndactyly, and retardation. Bell palsy is a demyelinating inflammatory process of the facial nerve that classically presents as an acute unilateral facial paralysis and is believed to be caused by the herpes simplex virus.
In the United States, occurrence of encephaloceles is most common in which of the following anatomic regions?
(A) Frontoethmoidal
(B) Occipital
(C) Parietal
(D) Sphenoidal
The correct response is Option B.
In the United States, encephaloceles are most common in the occipital location. In Asia, they are more common in the frontoethmoidal region. The prognosis with these malformations depends upon the presence or absence of herniated brain tissue in the encephalocele.
Which of the following genetic mutations is most likely to be found in a patient with orofacial clefting and popliteal pterygium?
A) Gain-of-function mutation in FGFR2
B) Gain-of-function mutation in NF1
C) Gain-of-function mutation in PIK3CA
D) Loss-of-function mutation in IRF6
E) Loss-of-function mutation in TCOF1
The correct response is Option D.
IRF6 mutations that result in loss-of-function have been reported in both syndromic Van der Woude syndrome as well as nonsyndromic orofacial clefting; Van der Woude syndrome can include popliteal pterygium.
PIK3CA gain-of-function mutations have been reported with venous malformations and lymphatic malformations. Mutations in TCOF1 are associated with Treacher Collins syndrome; these patients may have cleft palate, but they do not have lip pits. Gain-of-function mutations in FGFR2 have been implicated in syndromic craniosynostosis (Apert syndrome, Crouzon syndrome, and others). Mutations in NF1 result in increased RAS/MAPK signaling and neurofibromatosis type 1.
A 2-week-old male newborn is brought to the office for evaluation of median craniofacial dysraphia. His parents say that aside from the deformity, the child is thriving. Physical examination shows Tessier No. 0 and 14 clefts. A clinical photograph and a three-dimensional CT scan are shown. Which of the following additional findings is most likely in this patient?
A) Choanal atresia
B) Contact between the dura and ectoderm through the anterior fontanelle
C) Failure of closure of the foramen cecum
D) Heterotopic glial tissue
E) Rathke pouch cyst
The correct response is Option C.
OThe classification scheme by Tessier is perhaps the most accepted and basic approach to describe these rare craniofacial clefts. As it only provides an anatomical description, others have further developed description schemes. Some have an embryonic emphasis to further characterize these disorders. These severe clefts can be associated with encephalocele, which if not repaired can be life-threatening. Nasal dermoids arise from contact between dura and ectoderm through the foramen cecum. These can be dangerous if there is intracranial extension. Heterotopic glial tissue is equivalent to the term glioma because these are generally not connected to the intracranial space and are treated as benign lesions. The Rathke pouch occurs during development when the stomodeum ectoderm invaginates toward the hypophysis and remnants are usually located in the nasopharynx as a cyst. An encephalocele is similar to a glioma but contains meninges and/or brain (encephalomeningocele) and can communicate with the ventricle (encephalomeningocystocele). Fifteen percent of these are intranasal. Resection of these without knowing about their intracranial communication can be disastrous. Left untreated, these lesions can significantly increase the risk of meningitis and be life-threatening.
An otherwise healthy 5-year-old girl has a yellow, cystic mass on the left sclera, a left preauricular branchial cleft remnant, elevation of the left oral commissure, and soft-tissue deficiency of the left face. It is most appropriate to order which of the following tests to assess for significant comorbidities?
A) Echocardiography
B) Fluorescence in situ hybridization of the branchial cleft remnant
C) Funduscopic examination
D) MRI of the brain
E) Plain x-ray studies of the hands
The correct response is Option C.
The patient described has Goldenhar syndrome, a severe form of hemifacial microsomia with variable ear anomalies, mandibular hypoplasia leading to occlusal cant and oral commissure asymmetry, and soft-tissue deficiency on the affected side. Ocular findings associated with Goldenhar syndrome are variable and common (50% incidence) and can occur as epibulbar dermoids as described in this patient as well as microphthalmia, eyelid and optic nerve colobomas (interruption of a circular structure of the eye). Colobomas can be asymptomatic (affecting the iris alone), can lead to exposure keratopathy (in the case of eyelid colobomas), or can lead to visual disturbances (optic disc/nerve coloboma), such as visual field deficits or amblyopia. Care must be taken to identify visual disturbances early in order to implement contralateral eye penalization, or patching, to maximize visual development of the affected eye. The other tests target areas that are not affected in patients with Goldenhar syndrome.
A 20-month-old girl is brought to the office for evaluation of mid face hypoplasia, craniosynostosis, and anomalies of the hands and feet. The most likely cause of these findings is a genetic error in which of the following?
A) 22q.11
B) FGFR2
C) IRF6
D) TCOF1
The correct response is Option B.
The patient described has Apert syndrome. This autosomal dominant syndrome is characterized by bicoronal craniosynostosis leading to turribrachycephaly, mid face hypoplasia, and complex hand and foot syndactyly. Patients with Crouzon syndrome, an autosomal dominant disorder, typically have craniosynostosis involving the coronal, sagittal, and lambdoid sutures, as well as turribrachycephaly. Other findings include mid face hypoplasia, exorbitism, and proptosis. The extremities are normal. Defects in the Fibroblast Growth Factor Receptor-2 (FGFR2) are found.
22q deletion syndrome, which has several presentations, including DiGeorge syndrome, velocardiofacial syndrome, and Shprintzen syndrome, is caused by the deletion of a small piece of chromosome 22. The deletion occurs near the middle of the chromosome at a location designated q11.2; i.e., on the long arm of one of the pairs of chromosome 22. Characteristic signs include congenital heart disease, cleft palate, learning disabilities, mild elongation of facial features, and mental illness in the teenage years.
Mutations in the IRF6 gene cause van der Woude syndrome. Van der Woude syndrome is an autosomal dominant form of cleft lip and palate. Affected individuals usually have lip pits.
Mutations in the TCOF1 gene cause Treacher Collins syndrome. The official name of this gene is “Treacher Collins-Franceschetti syndrome 1.” Patients with Treacher Collins syndrome, or mandibular dysostosis, have hypoplasia of the zygoma, maxilla, and mandible, downward slanting of the palpebral fissures, colobomas of the lower eyelids, absence of eyelashes, and auricular defects.
A 3-month-old infant presents with scaphocephaly and sagittal suture ridging. Spring-assisted cranioplasty with an endoscopic approach is planned. Which of the following is an advantage of this procedure over open cranial vault remodeling procedures?
A) Decreased need for additional surgeries
B) Greater control of bony movements
C) Improved aesthetic outcomes
D) Less blood loss
E) Lower risk for developing neurocognitive impairment
The correct response is Option D.
Spring-assisted cranioplasty or spring-assisted synostosis surgery is a technique to address sagittal craniosynostosis. This procedure is similar in approach to suturectomy/helmet procedures, which involve removal of the fused suture via small incisions, often with the assistance of an endoscope. In spring-assisted cranioplasty, however, no helmet is applied to the skull to reshape it. Rather, two to three stainless steel springs are placed between the now separated parietal bones that are widened over time as the springs expand. These springs need to be removed around 4 months later in a short second operation. Compared with more traditional open cranial vault reconstruction procedures, spring-assisted cranioplasty is associated with decreased blood loss, shorter operative time (even including the second surgery) and hospital stay, and comparable aesthetic and neurocognitive outcomes. The surgeon is, however, compromising control of the postoperative outcome in favor of the smaller surgical footprint and the guaranteed second surgery required to remove the springs.